What is the hepatorenal syndrome? List causes, mechanism, diagnosis and treatment
The hepatorenal syndrome is defmed as profound oliguria and avid sodium retention in the setting of severe liver dysfunction.
{a) Causes:
Can occur in the setting of both acute and acute-on-chronic liver disease of almost any cause
eg. hepatitis, gestational liver failure, cirrl:tosis.
(b) Mechanism:
The pathogenesis appears to be purely functional in that recovery:of :liver function or-·
transplantation of the failing kidney leads to recovery of ·renal ·function. Mechanisms··· implicated include: hyperdynamic· circulation with lowering of renal perfusion pressure, activation of the sympathetic nervous system and a combination of precapillary vasoconstriction and past capillary dilatation by vasoactive mediators leading to decreased glomerular ultrafiltration coefficient. Hypovolaemia and raised intra abdominal pressure from ascites may also be factors.
(c) Diagnosis:
Is based on history (deteriorating renal function in the presence of severe liver disease) and the combination of avid sodiwn retention (UNa <30mmolll), oliguria, unremarkable urinalysis and sediment, absence of obstruction and exclusion of intravascular volume depletion.
(d) Treatment:
Classic teaching is that. in the absence of liver function recovery or liver transplantation,
there is no treatment other than renal replacement therapy for established hepatorenal
syndrome. Other measures to be considered or experimental are:
• Volume expansion and albumin infusion
• Paracentesis
• Relief of portal hypertension (TIPS, shunt)
• Vasopressin analogs (omipressin), experimental
• Prostaglandin analogs -experimental
Briefly, what is hepatorenal syndrome
Hepatorenal syndrome is kidney injury as the consequence of gradually reducing renal blood flow, in the context of gradually worsening liver failure.
Mechanism of hepatorenal syndrome
Causes of hepatorenal syndrome
Diagnosis
Treatment of hepatorenal syndrome
if all else fails, TIPS procedure may be appropriate.
UpToDate has an excellent synopsis of hepatorenal syndrome.
However, if you are cheap, you need free articles.
Salerno, Francesco, et al. "Diagnosis, prevention and treatment of hepatorenal syndrome in cirrhosis." Postgraduate medical journal 84.998 (2008): 662-670.
Solà, Elsa, Mónica Guevara, and Pere Ginès. "Current treatment strategies for hepatorenal syndrome." Clinical Liver Disease 2.3 (2013): 136-139.
A forty-two (42) year old man has been well, apart from a history of alcohol induced liver dysfunction and portal hypertension. He has abstained from alcohol for the past 8 months after being told that it would kill him. After a large haematemesis he presents drowsy, clinically shocked, with a blood pressure of 80 systolic, heart rate of 124 beats/minute, cold and clammy peripheries. He is also clinically jaundiced.
(a) Outline the principles of, and rationale for, the initial management of this patient.
Principles of management include resuscitation (of someone who may well have lost >25% of blood volume), establishing a diagnosis, and definitive treatment while avoiding therapies that might worsen his underlying condition.
Resuscitation includes assessment of airway protection and breathing adequacy, which combined with neurological impairment indicates need for emergency intubation.
Circulatory support requires adequate intravenous access, but may not need to be too aggressive (as excessive resuscitation may worsen portal hypertension), and could be guided by factors including usual blood pressure (? accept MAP of 60 mmHg), urine output, and other signs of circulatory compromise.
Temporary use of a Sengstaken Blakemore tube (or equivalent eg Minnesota) may be considered if blood loss is uncontrollable. Invasive monitoring may be useful, but is not necessary in the early phase of resuscitation.
Establishing a diagnosis for the cause of bleeding includes immediate examination (signs suggesting non-GI haemorrhage, chronic liver disease), history (from family/observers) of immediate event and possible precipitants (drugs, retching etc). Differential diagnosis of causes for jaundice should be considered (including hepatitic and toxins). More detailed history and examination will need to be completed later.
Immediate commencement of therapy (eg. intravenous vasoconstrictor such as somatostatin or vasopressin) should be considered while organising urgent endoscopy (which will usually result in banding or sclerotherapy for varices, and injection for some other pathologies). Initial investigations should include cross match, coagulation tests, full blood count, urea & electrolytes, liver function tests, blood glucose and paracetamol level. An ascitic tap should be undertaken early for microscopy and culture (as infection may well be present). Some rationale should have been given for the investigations listed.
Candidates should be aware of therapies that may be specifically required (correction of coagulopathy [FFP &/or platelets], prophylactic antibiotics, laxatives eg. lactulose; beta-blockers once stable, proton pump inhibitors) or contraindicated (sedatives worsening hepatic encephalopathy).
I have attempted to force the coherent college answer into a familiar system.
(a) Outline the principles of, and rationale for, the initial management of this patient.
Oh's Intensive Care manual: Chapter 42 (pp. 487) Acute gastrointestinal bleeding by Joseph JY Sung
arcia-Tsao, Guadalupe, and Jaime Bosch. "Management of varices and variceal hemorrhage in cirrhosis." New England Journal of Medicine 362.9 (2010): 823-832.
García-Pagán, Juan Carlos, et al. "Early use of TIPS in patients with cirrhosis and variceal bleeding." New England Journal of Medicine 362.25 (2010): 2370-2379.
Vlavianos, P., et al. "Balloon tamponade in variceal bleeding: use and misuse."BMJ: British Medical Journal 298.6681 (1989): 1158.
Reverter, Enric, and Juan Carlos García‐Pagán. "Management of an acute variceal bleeding episode." Clinical Liver Disease 1.5 (2012): 151-154.
Ioannou, G. N., J. Doust, and D. C. Rockey. "Terlipressin in acute oesophageal variceal haemorrhage." Alimentary pharmacology & therapeutics 17.1 (2003): 53-64.
Corley, Douglas A., et al. "Octreotide for acute esophageal variceal bleeding: a meta-analysis." Gastroenterology 120.4 (2001): 946-954.
Reiberger, Thomas, et al. "Carvedilol for primary prophylaxis of variceal bleeding in cirrhotic patients with haemodynamic non-response to propranolol." Gut62.11 (2013): 1634-1641.
Hou, Ming‐Chih, et al. "Antibiotic prophylaxis after endoscopic therapy prevents rebleeding in acute variceal hemorrhage: a randomized trial." Hepatology 39.3 (2004): 746-753.
A forty-two (42) year old man has been well, apart from a history of alcohol induced liver dysfunction and portal hypertension. He has abstained from alcohol for the past 8 months after being told that it would kill him. After a large haematemesis he presents drowsy, clinically shocked, with a blood pressure of 80 systolic, heart rate of 124 beats/minute, cold and clammy peripheries. He is also clinically jaundiced.
(b) Variceal bleeding is diagnosed and it initially responds to therapy.
48 hours post admission he remains on invasive respiratory support, with weak withdrawal response to pain despite minimal sedation, a persistent coagulopathy, and is inotrope dependent. Serum bilirubin concentration is elevated (100 micromol/L [N 3-20]).
He develops a further acute variceal bleed associated with hypotension.
Outline your management of this episode.
Standard resuscitation goals and technique should be reiterated. Re-bleeding from varices requires repeat endoscopy for diagnosis and treatment. Additional treatments should be considered including vasoconstrictor infusions (eg. somatostatin or vasopressin with GTN), Trans-jugular Intra-hepatic Porto-systemic Shunt (TIPS), and surgical shunts (eg. spleno-renal). Balloon tamponade is being used less frequently because of a high incidence of complications (aspiration, oesophageal rupture, death).
Ongoing investigation and treatment of coagulopathy, and investigation of causes of jaundice should be undertaken. Treatment should include strategies to minimise hepatic encephalopathy.
A systematic approach should be taken. I will not repeat the ABCs. Of course, one would ensure satisfactory maintenance of oxygenation and normotension. Certainly, one would replace the missing factors by transfusing blood products, as well as actual packed cells, and vitamin K.
Straight to the specific management:
Garcia-Tsao, Guadalupe, and Jaime Bosch. "Management of varices and variceal hemorrhage in cirrhosis." New England Journal of Medicine 362.9 (2010): 823-832.
García-Pagán, Juan Carlos, et al. "Early use of TIPS in patients with cirrhosis and variceal bleeding." New England Journal of Medicine 362.25 (2010): 2370-2379.
Vlavianos, P., et al. "Balloon tamponade in variceal bleeding: use and misuse."BMJ: British Medical Journal 298.6681 (1989): 1158.
Reverter, Enric, and Juan Carlos García‐Pagán. "Management of an acute variceal bleeding episode." Clinical Liver Disease 1.5 (2012): 151-154.
Ioannou, G. N., J. Doust, and D. C. Rockey. "Terlipressin in acute oesophageal variceal haemorrhage." Alimentary pharmacology & therapeutics 17.1 (2003): 53-64.
Corley, Douglas A., et al. "Octreotide for acute esophageal variceal bleeding: a meta-analysis." Gastroenterology 120.4 (2001): 946-954.
Reiberger, Thomas, et al. "Carvedilol for primary prophylaxis of variceal bleeding in cirrhotic patients with haemodynamic non-response to propranolol." Gut62.11 (2013): 1634-1641.
A forty-two (42) year old man has been well, apart from a history of alcohol induced liver dysfunction and portal hypertension. He has abstained from alcohol for the past 8 months after being told that it would kill him. After a large haematemesis he presents drowsy, clinically shocked, with a blood pressure of 80 systolic, heart rate of 124 beats/minute, cold and clammy peripheries. He is also clinically jaundiced.
Variceal bleeding is diagnosed and it initially responds to therapy. 48 hours post admission he remains on invasive respiratory support, with weak withdrawal response to pain despite minimal sedation, a persistent coagulopathy, and is inotrope dependent. Serum bilirubin concentration is elevated (100 micromol/L [N 3-20]). He develops a further acute variceal bleed associated with hypotension.
(c) At 6 days there has been no further haematemeses. However he has a Glasgow Coma Score (GCS) of 5, despite no sedation. His serum bilirubin concentration is now 350 micromol/L. Prothrombin time and serum creatinine concentration are twice normal. A CT of the head shows no focal abnormality. What supportive therapies and strategies would you have in place at this stage and why?
Specific strategies to minimise hepatic encephalopathy should have been described if not already done so (including the use of lactulose). Precipitants must be minimised (treatment of infections, avoidance of sedatives, correction of electrolyte abnormalities/hypoxia, avoid alkalosis, limit dietary protein, consider unproven dietary supplements including BCAA etc.). Cautious volume expansion should be considered. Other reversible causes for renal dysfunction and coma should be sought and excluded. Management of ICP may be necessary (and the CT does not exclude cerebral oedema). General supportive care should be considered (eg. physiotherapy, avoidance of line- related problems, family support etc.). Specific treatment may be required for ascites and its effects (drainage, colloid replacement etc).
The college has given us a patient with a combination of several reasons to be encephalopathic; of these, the major one is liver failure -but we are also reminded that the creatinine has doubled.
Management of hepatic encephalopathy:
Specific management of hepatic encephalopathy
Management of the precipitating cause
Supportive management of the encephalopathic patient
Pursuit of other explanations for decreased level of consciousness:
Wendon, Julia, et al. "Critical care and cirrhosis: outcome and benefit." Current opinion in critical care 17.5 (2011): 533-537.
Riggio, Oliviero, et al. "Management of hepatic encephalopathy as an inpatient." Clinical Liver Disease 5.3 (2015): 79-82.
Bajaj, J. S. "Review article: the modern management of hepatic encephalopathy." Alimentary pharmacology & therapeutics 31.5 (2010): 537-547.
Amodio, Piero, et al. "The nutritional management of hepatic encephalopathy in patients with cirrhosis: International Society for Hepatic Encephalopathy and Nitrogen Metabolism Consensus." Hepatology 58.1 (2013): 325-336.
Als-Nielsen, Bodil, Lise Lotte Gluud, and Christian Gluud. "Nonabsorbable disaccharides for hepatic encephalopathy." Cochrane Database Syst Rev 2 (2004).
Bass, Nathan M., et al. "Rifaximin treatment in hepatic encephalopathy." New England Journal of Medicine 362.12 (2010): 1071-1081.
A forty-two (42) year old man has been well, apart from a history of alcohol induced liver dysfunction and portal hypertension. He has abstained from alcohol for the past 8 months after being told that it would kill him. After a large haematemesis he presents drowsy, clinically shocked, with a blood pressure of 80 systolic, heart rate of 124 beats/minute, cold and clammy peripheries. He is also clinically jaundiced.
Variceal bleeding is diagnosed and it initially responds to therapy. 48 hours post admission he remains on invasive respiratory support, with weak withdrawal response to pain despite minimal sedation, a persistent coagulopathy, and is inotrope dependent. Serum bilirubin concentration is elevated (100 micromol/L [N 3-20]). He develops a further acute variceal bleed associated with hypotension.
At 6 days there has been no further haematemeses. However he has a Glasgow Coma Score (GCS) of 5, despite no sedation. His serum bilirubin concentration is now 350 micromol/L. Prothrombin time and serum creatinine concentration are twice normal. A CT of the head shows no focal abnormality.
(d) His wife tells you that he had been recently unjustly fired from work and for the week prior to his admission had started to drink heavily again. He had complained of headache for which he would frequently take paracetamol and had been eating poorly. She asks you what are his chances of survival. How do you respond?
Prognosis of hepatic encephalopathy and associated organ dysfunction depends on whether the process is acute or chronic, and whether there are any reversible factors. The very high bilirubin level (350 mcgmol/L), and the fact that this man has rebled from his varices make his prognosis worse, but not unsalvageable. Shunting procedures may decrease his likelihood of further bleeding but are likely to worsen the encephalopathy.
Severe hepatic encephalopathy in ICU seems to actually have a slightly better outcome than other sorts of organ system failures.
Several things can be added, with regard to prognostication in chronic liver disease patients admitted to ICU:
Fichet, Jérôme, et al. "Prognosis and 1-year mortality of intensive care unit patients with severe hepatic encephalopathy." Journal of critical care 24.3 (2009): 364-370.
García-Martínez, Rita, Macarena Simón-Talero, and Juan Córdoba. "Prognostic assessment in patients with hepatic encephalopathy." Disease markers 31.3 (2011): 171-179.
Wendon, Julia, et al. "Critical care and cirrhosis: outcome and benefit." Current opinion in critical care 17.5 (2011): 533-537.
Fichet, Jérôme, et al. "Prognosis and 1-year mortality of intensive care unit patients with severe hepatic encephalopathy." Journal of critical care 24.3 (2009): 364-370.
García-Martínez, Rita, Macarena Simón-Talero, and Juan Córdoba. "Prognostic assessment in patients with hepatic encephalopathy." Disease markers 31.3 (2011): 171-179.
Wendon, Julia, et al. "Critical care and cirrhosis: outcome and benefit." Current opinion in critical care 17.5 (2011): 533-537.
Pugh, R. N. H., et al. "Transection of the oesophagus for bleeding oesophageal varices." British Journal of Surgery 60.8 (1973): 646-649.
Forman, Lisa M., and Michael R. Lucey. "Predicting the prognosis of chronic liver disease: an evolution from Child to MELD." Hepatology 33.2 (2001): 473-475.
Child, CG_, and J. G. Turcotte. "Surgery and portal hypertension." Major problems in clinical surgery 1 (1964): 1.
Malinchoc, Michael, et al. "A model to predict poor survival in patients undergoing transjugular intrahepatic portosystemic shunts." Hepatology 31.4 (2000): 864-871.
Zimmerman, Jack E., et al. "Intensive care unit admissions with cirrhosis: Risk‐stratifying patient groups and predicting individual survival." Hepatology 23.6 (1996): 1393-1401.
Cholongitas, E., et al. "Systematic review: the model for end‐stage liver disease–should it replace Child‐Pugh's classification for assessing prognosis in cirrhosis?." Alimentary pharmacology & therapeutics 22.11‐12 (2005): 1079-1089.
Ho, Yu-Pin, et al. "Outcome prediction for critically ill cirrhotic patients: a comparison of APACHE II and Child-Pugh scoring systems." Journal of intensive care medicine 19.2 (2004): 105-110.
Cholongitas, E., et al. "Risk factors, Sequential Organ Failure Assessment and Model for End‐stage Liver Disease scores for predicting short term mortality in cirrhotic patients admitted to intensive care unit." Alimentary pharmacology & therapeutics 23.7 (2006): 883-893.
Levesque, Eric, et al. "Prospective evaluation of the prognostic scores for cirrhotic patients admitted to an intensive care unit." Journal of hepatology 56.1 (2012): 95-102.
Yeoh, Sern Wei, et al. "Cirrhotics Treated In Intensive Care Unit Have High Short Term Survival in the Absence of Extrahepatic Organ Dysfunction." Journal of Gastroenterology and Hepatology Research 5.2 (2016): 1984-1988.
Wiesner, Russell, et al. "Model for end-stage liver disease (MELD) and allocation of donor livers." Gastroenterology 124.1 (2003): 91-96.
Saliba, Faouzi, et al. "Cirrhotic patients in the ICU: prognostic markers and outcome." Current opinion in critical care 19.2 (2013): 154-160.
Peng, Ying, Xingshun Qi, and Xiaozhong Guo. "Child–Pugh Versus MELD Score for the Assessment of Prognosis in Liver Cirrhosis: A Systematic Review and Meta-Analysis of Observational Studies." Medicine 95.8 (2016).
List the information that can be obtained from ascitic fluid analysis. What are your indications for an ascitic fluid tap?
Ascitic fluid analysis provides – Fluid for General appearance
Albumin / protein content
Red cell count White cell count Culture and sensitivity Cytology
Biochemistry - amylase
Indications for performing a tap include: any patient with ascites and PUO, critical illness or suspected malignancy.
I would expand the college list:
Information which can be derived from an ascitic fluid analysis:
Indications for ascitic fluid drainage:
Bar-Meir, Simon, Emanuel Lerner, and Harold O. Conn. "Analysis of ascitic fluid in cirrhosis." Digestive diseases and sciences 24.2 (1979): 136-144.
Boyer, Thomas D., Arthur M. Kahn, and Telfer B. Reynolds. "Diagnostic value of ascitic fluid lactic dehydrogenase, protein, and WBC levels." Archives of internal medicine 138.7 (1978): 1103-1105.
Runyon, Bruce A., John C. Hoefs, and Timothy R. Morgan. "Ascitic fluid analysis in malignancy‐related ascites." Hepatology 8.5 (1988): 1104-1109.
Critically evaluate the use of cisapride, metoclopramide and erythromycin for gastric emptying in Intensive Care patients.
Cisapride: selectively enhances physiologic release of acetylcholine at level of myenteric plexus. Part of effect via activation of serotonin (5-HT4) receptors. Enhances oesophageal peristaltic activity, gastric emptying, intestinal propulsive activity and colonic transit. Extensively metabolised via cytochrome P450 3A4 enzymes. Highly protein bound. Only administered orally. Significant adverse effects and interactions, especially prolonged QT interval (and arrhythmias) in particular when administered in patients at risk of arrhythmias or when administered concurrently with drugs that prolong QT or drugs that inhibit P450 3A4 enzymes (e.g. azole antifungals, macrolide antibiotics, and protease inhibitors). Problems with limited availability, restrictions on prescribing, large number of documented interactions.
Metoclopramide: mode of action unclear (? via selective dopamine-2 receptor antagonist effects); sensitises tissues to the action of acetylcholine (motility effects abolished by anticholinergic drugs and narcotic analgesics). Increases tone and amplitude of gastric contractions, relaxes pyloric sphincter and increases peristalsis of duodenum and jejunum. Administered orally, IV or IM. Conjugated by liver and renally excreted (reduced clearance with renal failure). Minimal protein binding. Dopamine agonist activity responsible for adverse effects (e.g. sedation, dystonic/extrapyramidal reactions).
Erythromycin: macrolide antibiotic that seems to stimulate motilin receptors, and enhances motilin release from enterochromaffin cells of duodenum. Enhanced contractile effects on gastric antrum and duodenum. Administered orally or intravenously ( probably IV more effective). Highly protein bound. Substantial hepatic metabolism. Prolonged QT and arrhythmias reported, as have hepatic dysfunction, overgrowth of non-susceptible organisms and colitis (Cl. difficile). Elevated levels of many other drugs (as a result of inhibition of metabolism) can lead to toxicity (e.g. theophylline, HMG-CoA reductase inhibitors, anti-epileptics, digoxin, warfarin etc).
One study ran all of these drugs against each other to evaluate their comparative efficacy. The only useful finding was that metoclopramide worked faster than cisapride. And then cisapride was withdrawn from the market, following concerns of toxicity. The current Canadian Critical Care Nutrition Guidelines only mention metoclopramide and erythromycin (as well as enteral naloxone).
Features |
Metoclopramide |
Erythomycin |
Cisapride |
Class and mechanism |
Antiemetic; Dopamine receptor antagonist Enhances gastric emptying rate and increases the tone of the oeseophageal sphincter |
Macrolide antibiotic; Motilin receptor agonist, motilin release enhancer Increases the automaticity of enteric nervous system motor function |
Prokinetic 5-HT4 receptor agonist Enhances oesophageal peristaltic activity, gastric emptying, intestinal propulsive activity and colonic transit (although in the wake of its discontinuation, many believe these effects were overstated). |
Advantages |
Low toxicity |
Low toxicity |
None? |
Adverse effects |
Increased prolactin release |
Allergic reactions |
QT prolongation |
MacLaren, Robert, et al. "Sequential single doses of cisapride, erythromycin, and metoclopramide in critically ill patients intolerant to enteral nutrition: a randomized, placebo-controlled, crossover study." Critical care medicine 28.2 (2000): 438-444.
The best resource for all EBM in this topic is the Canadian Critical Care Nutrition Guidelineswebsite.
Society Of Critical Care Medicine and American Society for Parenteral and Enteral Nutrition.Guidelines for the provision and assessment of nutrition support therapy in the adult critically ill patient. Crit Care Med 2009 Vol. 37, No. 5 , 2009
Critically evaluate the strategies for prevention of gastrointestinal bleeding in the critically ill.
Answers should address more than just prevention of gastric erosions/stress ulceration.
Consideration should be given to other causes including patients with known gastro-oesophageal varices (where sclerotherapy/banding, beta-blockers and techniques to lower venous pressure, and avoidance of local trauma should be considered).
With regard to stress ulceration many strategies have been employed, and should be considered in a broad answer. General resuscitation of patients, correction of coagulopathy, early enteral feeding and avoidance of precipitants (eg. NSAIDs) in patients at risk are assumed to be beneficial (but not well studied). Prospective randomised trials have generally compared drug regimens (antacids vs sucralfate vs H2-blockers vs proton pump inhibitors). Other agents include prostaglandin analogs. Controversy surrounds the issues of widespread use of prophylactic agents, value of drugs vs placebo, nosocomial infection rates, and cost-benefit analyses.
"Critically evaluate" demands a certain degree of structure. Contrary to the college answer, banding and sclerotherapy are not usually preventative techniques. If one were to dedicate any time during this ten minute answer to varices and leaking AVMs, one may wish to be very brief about it, as the meat is clearly in gastric ulceration.
Introduction
Gastrointestinal bleeding in the critically ill patient may be due to a variety of causes; these include bleeding from stress ulceration, oesophageal varices, and colonic polyps. Exacerbating causes include antiplatelet and anticoagulant medications, as well as poor perfusion of gastrointestinal mucosa in the context of shock. Given that in the ICU GI bleeding is combined with a series of other major organ dysfunction syndromes, it tends to have a castarophic mortality rate and it is important to be able to protect at-risk patients from this complication.
Rationale
Options:
Advantages
Disadvantages
Evidence for the routine use of ulcer prophylaxis
Evidence to support one drug class over another
Summary
PPIs are indicated in at-risk patient in ICU who are intolerant of eneteral feeding, and who are otherwise at risk of gastrointestinal bleeding. Further research is required to discriminated between different classes of drugs in terms of efficacy, and to identify the at-risk population.
Steinberg, Kenneth P. "Stress-related mucosal disease in the critically ill patient: risk factors and strategies to prevent stress-related bleeding in the intensive care unit." Critical care medicine 30.6 (2002): S362-S364.
Marik, Paul E., et al. "Stress ulcer prophylaxis in the new millennium: a systematic review and meta-analysis." Crit Care Med 38.11 (2010): 2222-2228.
Krag, Mette, et al. "Stress ulcer prophylaxis versus placebo or no prophylaxis in critically ill patients." Intensive care medicine 40.1 (2014): 11-22.
Madsen, Kristian Rørbæk, et al. "Guideline for Stress Ulcer Prophylaxis in the Intensive Care Unit." Danish medical journal 61.3 (2014): 1-4.
Alshamsi, Fayez, et al. "Efficacy and safety of proton pump inhibitors for stress ulcer prophylaxis in critically ill patients: a systematic review and meta-analysis of randomized trials." Critical Care 20.1 (2016): 120.
Barletta, Jeffrey F., et al. "Stress ulcer prophylaxis." Critical care medicine44.7 (2016): 1395-1405.
List the potential aetiology of a severely altered mental status in a 65-year-old man, 48 hours after major hepatic resection for hepatocellular carcinoma. Outline your management of this patient.
Mental state could be severely depressed or patient may be agitated or confused.
In general the potential aetiologies are the same, though some more likely in each type of state. Consider: decreased oxygen delivery to braiin (hypoxaemia, low cardiac output, low blood pressure), effects of drugs (those administered or those withdrawing from), intracerebral pathology (thromboembolism, rarely bleed eg. into undetected secondaries), electrolyte disorders (especially glucose, Na and Calcium), infections (unlikely; eg. systemic/meningitis/encephalitis), postoperative confusional state (uncertain but probably multifactorial aetiology), post-ictal or psychiatric disorder.
Management involves exclusion of reversible and specific treatable causes considered likely/possible (eg. SpO2, vital signs, glucose, electrolytes, review drugs and history). Appropriate treatment of any specific abnormalities detected. Protection of patient and staff with cautious use of restraint (chemical or physical) if absolutely necessary or specifically indicated.
Potential aetiology for a severely altered mental status post hepatic resection:
Management:
Specific management of hepatic encephalopathy
Management of the precipitating cause
Supportive management of the encephalopathic patient
Wendon, Julia, et al. "Critical care and cirrhosis: outcome and benefit." Current opinion in critical care 17.5 (2011): 533-537.
Riggio, Oliviero, et al. "Management of hepatic encephalopathy as an inpatient." Clinical Liver Disease 5.3 (2015): 79-82.
Bajaj, J. S. "Review article: the modern management of hepatic encephalopathy." Alimentary pharmacology & therapeutics 31.5 (2010): 537-547.
Amodio, Piero, et al. "The nutritional management of hepatic encephalopathy in patients with cirrhosis: International Society for Hepatic Encephalopathy and Nitrogen Metabolism Consensus." Hepatology 58.1 (2013): 325-336.
Als-Nielsen, Bodil, Lise Lotte Gluud, and Christian Gluud. "Nonabsorbable disaccharides for hepatic encephalopathy." Cochrane Database Syst Rev 2 (2004).
Bass, Nathan M., et al. "Rifaximin treatment in hepatic encephalopathy." New England Journal of Medicine 362.12 (2010): 1071-1081.
A 60-year-old woman has a right hemi-hepatectomy for invasive cholangio-carcinoma. She has been admitted to your unit for postoperative care
a) Describe in detail what problems she may develop in the first 48 hours and how you would treat them?
The perioperative complications could be classified into (1) that of any major upper abdominal surgery and (2) specifically that of a hemi-hepatectomy for cholangiocarcinoma; or divided into various systems, ie.
(1) Respiratory: Inadequate or excessive analgesia, pulmonary oedema from fluid overload, R. haemothorax, R. pneumothorax, R diaphragmatic dysfunction, V/Q mismatch from hepatic failure, aspiration and possibly early pulmonary infection or thromboembolism. Very rarely, intraoperative air embolism ®ARDS.
(2) Cardiovascular: Hypotension from bleeding, epidural block, perioperative myocardial ischaemia / infarction, Arrhythmias associated with electrolyte abnormalities.
(3) Gastro-intestinal failure: Prolonged ileus, pseudo-obstruction, ascites, G I haemorrhage. (4) Renal: Hepatorenal syndrome, acute tubular necrosis, oliguria.
(5) Hepatic: Cholangitis, hepatic failure, encephalopathy, coagulopathy, (6) CNS: Encephalopathy.
(7) Metabolic: hyperlactataemia, iNa+, lK+, hypoglycaemia.
(8) Premorbid condition: Possible ulcerative colitis/primary sclerosing cholangitis: Therefore, medication issues ie steroids, immune state, nutritional status etc.
Treatment is basically meticulous perioperative care with special regard to fluid and electrolyte balance, analgesia, coagulation control, and specific and supportive therapy for any individual complications that develop ie encephalopathy, hepatorenal syndrome etc.
This question closely resembles Question 1 from the second paper of 2006. In the interest of simplified revision, the answer to that question is reproduced below:
Problems | Solutions |
Airway issues | Extubate them in ICU (no difference in duration of ICU stay, regardless of where they are extubated) (Neelakanta et al, 1997). |
Atelectasis | Use NIV (reintubation rates will be improved- Narita et al, 2010) |
Bleeding | Use a low CVP strategy (2-5mmHg) Intraoperatively, insist on occlusive manoeuvres (eg. Pringle manoeuvre) |
Analgesia | Remember the impaired clearance. Avoid benzodiazepines and long-acting opiates. Epidural seems to increase fluid requirements: use PCA instead. |
Delirium | Hepatic encephalopathy may develop if the patient had abnormal liver function preoperatively. One is referred to the chapter on hepatic encephalopathy for details of its management (spoiler: a lot of lactulose is involved) |
High lactate | Expect it. Unlikely to be related to real shock (more a reflection of poor residual liver function) Observe it. |
Low phosphate | Expect it. Replace it. The phosphate is being absorbed by the regenerating liver. |
Renal failure | In advanced cirrhosis, may represent hepatorenal syndrome. In other situations, it may be pre-renal (i.e. due to inadequate fluid resuscitation) or due to intraoperative renal vascular or ureteric injury. |
Fluid overload | Use concentrated colloids to maintain intravascular volume (eg. 20% albumin). |
Hypercatabolic state | Early enteral nutrition (not much benefit from TPN - Hotta et al, 2002) Optimal pre-operative nutrition is important. Branched-chain amino acids (BCAAs) should be mentioned, in spite of the fact that tey are probably pointless in this setting. |
Hypoglycaemia | Attentive BSL monitoring |
Coagulopathy | Attentive coag monitoring; likely no need for correction unless there is active bleeding |
Infection | Mainly in the setting of bile leaks, abdominal collections, VAP or line-related sepsis. In any case, broad-spectrum cover will be deployed, some combination of an extended-spectrum β-lactam and lactamase inhibitor, eg. Tazocin. |
Jarnagin, William R., et al. "Improvement in perioperative outcome after hepatic resection: analysis of 1,803 consecutive cases over the past decade." Annals of surgery 236.4 (2002): 397-407.
Page, Andrew J., and David A. Kooby. "Perioperative management of hepatic resection." Journal of gastrointestinal oncology 3.1 (2012): 19-27.
Wrighton, Lindsay J., et al. "Postoperative management after hepatic resection." Journal of gastrointestinal oncology 3.1 (2012): 41-47.
Thorat, Ashok, and Wei-Chen Lee. Critical Care Issues After Major Hepatic Surgery. INTECH Open Access Publisher, 2013.
Pagano, Duilio, et al. "The unreliability of continuous postoperative lactate monitoring after extended hepatectomies: single center experience." Updates in surgery 67.1 (2015): 33-37.
Ciuni, Roberto, et al. "Nutritional aspects in patient undergoing liver resection." Updates in surgery 63.4 (2011): 249-252.
Hotta, Tsukasa, et al. "Evaluation of postoperative nutritional state after hepatectomy for hepatocellular carcinoma." Hepato-gastroenterology 50.53 (2002): 1511-1516.
Richter, B., et al. "Nutritional support after open liver resection: a systematic review." Digestive surgery 23.3 (2006): 139-145.
Marchesini, Giulio, et al. "Nutritional supplementation with branched-chain amino acids in advanced cirrhosis: a double-blind, randomized trial." Gastroenterology 124.7 (2003): 1792-1801.
Kim, Say-June, Dong-Goo Kim, and Myung Duk Lee. "Effects of branched-chain amino acid infusions on liver regeneration and plasma amino acid patterns in partially hepatectomized rats." Hepato-gastroenterology 58.109 (2010): 1280-1285.
Neelakanta, Gundappa, et al. "Early tracheal extubation after liver transplantation." Journal of cardiothoracic and vascular anesthesia 11.2 (1997): 165-167.
Narita, Masato, et al. "Noninvasive ventilation improves the outcome of pulmonary complications after liver resection." Internal Medicine 49.15 (2010): 1501-1507.
Outline the diagnostic features, complications and treatment of critically ill patients with pancreatitis.
This is a complex field with a large amount of literature to collate. Pancreatitis is usually presents with persistent upper abdominal pain, associated with nausea and vomiting, which can be associated with signs of local tenderness through to peritonism, and/or signs of a systemic inflammatory response (e.g. fever, tachycardia) or signs of associated disorders (e.g. jaundice with biliary obstruction) or rarely signs of complications (e.g. ecchymotic discoloration in flank [Grey-Turner’s sign] or peri-umbilical [Cullen’s sign] regions). These signs may be difficult to elicit or masked in critically ill patients. Investigations that assist in the diagnosis include: serum amylase (usually > 3 times normal) (serum lipase does not improve diagnostic accuracy); liver function tests (looking for evidence of obstructive pattern with gall stone induced pancreatitis); plain abdominal radiograph (excludes other aetiologies, and may show localised ileus [“sentinel loop”]; abdominal ultrasound (enlarged hypo-echoic pancreas, and looking for gall stones); and abdominal CT scan with contrast (confirm diagnois and looking for areas of necrosis or pseudocysts). Ranson’s criteria (or more recently Glasgow criteria or Imrie score) are used to assess severity and predict outcome, and they include white cell count (>16,000/mm3), glucose (>11 mmol/L), AST > 250 IU/L, Ca < 2mmol/L, hypoxaemia (<8kPa), and a decrease in haematocrit (>10%) and an increase in urea (>1.8 mmol/L).
Complications include: those associated with a systemic inflammatory response (e.g. myocardial depression/shock, ARDS, renal failure, death); respiratory (including pleural effucsion and atelectasis); metabolic (including hypocalcaemia, glucose disturbances); and intrabdominal problems (including ileus, necrosis, pseudo-cysts, abscess formation, etc).
Treatment should include: aggressive fluid resuscitation to stabilise the haemodynamic state, treatment of underlying cause (e.g. ERCP if gall stones present, withdrawal of offending drug), treatment of pain (morphine controversial), surgical treatment of complications (e.g. aspiration/drainage of infected collections) and general support of the critically ill patient. More contentious issues that should be considered include: early prophylactic broad spectrum antibiotics (evidence that decrease complications), prophylactic anti-fungal therapy, jejunal feeding (safe, feasible, cheaper than TPN, possibly of benefit), the use of somatostatin, octreotide or protease inhibitors (none have sufficient evidence base to use routinely), and the timing and nature of surgical interventions.
This question dates back to a time when there was a whole massive period during which one had plenty of thinking room, planning, and then the process of written communication could take place at a civilised pace, without rush. Now, of course, a question like this has the candidate trying to fit the last 30 years of pancreatitis literature into a ten minute answer.
One patently cannot do this.
The suggested answer below is one which could easily be produced over the course of ten minutes.
Diagnostic features
Complications
Treatment
Wilmer, Alexander. "ICU management of severe acute pancreatitis." European journal of internal medicine 15.5 (2004): 274-280.
A 45-year-old intellectually handicapped man is admitted to your Intensive Care Unit for airway management. He was nasally intubated for evacuation of a large dental abscess, which had caused airway compromise.
(c) Over the next 48 hours he develops increasing jaundice, with severe derangement of his Liver Function Tests. What are the likely causes, and how are you going to manage this problem?
(c) Over the next 48 hours he develops increasing jaundice, with severe derangement of his Liver Function Tests. What are the likely causes, and how are you going to manage this problem?
The potential causes of jaundice and abnormal LFTs within the first 72 hours are many. The pattern of elevation may help the diagnosis (eg. hepatocellular pattern [elevated transaminases, but minor elevation of Alkaline Phosphatase], cholestatic [minor elevation of transaminases]), and a systematic approach is helpful. Most likely causes include infection (systemic sepsis, mild hepatitic/intravascular cholestasis, liver abscess, acalculous cholecystitis), drug induced (cholestatic/hepatitic), haemodynamic/shock (ischaemic hepatitis) or haemolysis (sepsis, early destruction of transfused blood). Pre-existing intercurrent diseases (hepatitis, gall stones) could also be present.
Management depends on the specific/likely aetiology. A careful history (including drug history [eg. high dose of paracetamol before presentation]) and clinical examination (eg. signs of right heart failure, chronic liver disease, abdominal pain) followed by specific liver function tests to delineate the pattern of abnormality (including alkaline phosphatase [AP], gamma glutamyl transpeptidase [GGT] and/or conjugated/unconjugated bilirubin). More specific blood tests may be indicated (eg. haemolysis screen or viral serology). Imaging of right upper quadrant with ultrasound (to assess obstruction &/or stones) would usually be indicated (± other imaging eg. nuclear medicine or CT scan). After addressing the specific aetiology, further treatment would be largely supportive (with awareness of effects on drug metabolism).
The college has cheated the candidate by not presenting them with a list of LFTs to analyse. With no information, the differentials (and thus the manaement options) are distrubingly broad.
One can work though this systematically.
The following tests will need to be ordered, in order of escalating expense, invasiveness and esotericims:
Differentials and their management
In addition, one would need to adjust drug doses and dosing intervals to allow for changes in hepatic clearance.
These are the biochemical results taken from a 50-year-old woman, missing from an alcohol rehabilitation programme, who was found in her home by police three days since she was last seen. Blood was drawn for investigation.
Na |
126 |
mmol/l |
138 - 145 |
K |
3.5 |
mmol/l |
3.6 - 5.2 |
Cr |
0.25 |
mmol/l |
0.04 - 0.1 |
Urea |
7.0 |
mmol/l |
3.1 - 7.5 |
Bilirubin (Total) |
509 |
micromol/l |
2 - 22 |
Protein (Total) |
40 |
g/l |
65 - 85 |
Albumin |
20 |
g/l |
38 - 48 |
ALP |
153 |
IU/L |
40 - 100 |
GGT |
459 |
IU/L |
0 - 50 |
ALT |
336 |
IU/L |
0 - 45 |
CK |
400 |
IU/L |
30 - 180 |
TroponinT |
0.1 |
mcg/l |
0.00 - 0.03 |
Glucose |
3.2 |
mmol/l |
3.5 - 5.6 |
Ammonia |
342 |
micromol/l |
0 - 50 |
Lactate |
3.7 |
mmol/l |
0.6 - 2.4 |
Based on these results, what is the most likely cause of her reduced conscious state? Give reasons. What other simple blood test would support this diagnosis? What is the significance of the high plasma creatinine but normal urea concentrations?
Most likely cause is hepatic encephalopathy, but many other conditions would be excluded as part of her work up. She has obvious liver dysfunction (GGT, ALT, bilirubin), supported by a low albumin and a very high ammonia level. Other results (eg. Na, Cr/urea, glucose) are not as extreme and less likely to contribute to her reduced conscious state.
An additional assessment of the synthetic function of the liver would support the diagnosis(eg. prothrombin time, which is a marker of severity and should be prolonged).
The high plasma creatinine probably reflects significant renal impairment (not rhabdomyolysis as CK only mildly elevated). The urea value is probably a reflection of decreased production and a nutritional deficiency. A normal value makes gastrointestinal bleeding and severe hypovolaemia unlikely as precipitants for the encephalopathy.
This question closely resembles (but is not identical to) Question 29.2 from the second paper of 2011. The key difference is that in 2011, it was a 48 yr old male who escaped from the alcohol rehab camp. However, the bloods were identical.
In contrast to the 2011 question, this time the examiners are asking us for ONE diagnosis.
So, if the candidate were to put all their money on one pony, it would be hepatic encephalopathy. The ammonia, bilirubin, LFTs and background history all point to this. The one test which the college has not provided us with is the coags- coagulopathy would be informative as a part of the staging process for this condition, particularly the prothrombin time.
The significance of the creatitine and urea disparity is probably an issue of dehydration. The patient, haveing collapsed some time ago, has spent some hours/days slowly dehydrating. However, owing to the greatly diminished oral intake and muscle mass, the patient probably has a very slow rate of urea production. The absence of CK demonstrates the fact that rhabdomyolysis has no role to play in this process.
A 60 year old woman has a right hemi-hepatectomy for invasive cholangio- carcinoma. She has been admitted to the Intensive Care Unit for postoperative care.
List the problems she may develop in the first 48 hours.
The perioperative complications could be classified into (1) that of any major upper abdominal surgery and (2) specifically that of a hemi-hepatectomy for cholangiocarcinoma; or divided into various systems, i.e.
(1) Respiratory: Inadequate or excessive analgesia, pulmonary oedema from fluid overload, R. haemothorax, R. pneumothorax, R diaphragmatic dysfunction, V/Q mismatch from hepatic failure, aspiration and possibly early pulmonary infection or thromboembolism. Very rarely, intraoperative air embolism ®ARDS.
(2) Cardiovascular: Hypotension from bleeding, epidural block, perioperative myocardial ischaemia / infarction, Arrhythmias associated with electrolyte abnormalities.
(3) GI failure: Prolonged ileus, pseudo-obstruction, ascites, G I haemorrhage
(4) Renal: Hepatorenal syndrome, acute tubular necrosis, oliguria.
(5) Hepatic: Cholangitis, hepatic failure, encephalopathy, coagulopathy,
(6) CNS: Encephalopathy.
(7) Metabolic: hyperlactataemia, low Na+, high K+, hypoglycaemia.
(8) Premorbid condition possible ulcerative colitis/primary sclerosing cholangitis: Therefore, medication issues i.e. steroids, immune state, nutritional status etc.
This is a question which benefits from a systematic approach. The college answer has already made this attempt.
Thus: a table of Problems and Solutions:
Problems | Solutions |
Airway issues | Extubate them in ICU (no difference in duration of ICU stay, regardless of where they are extubated) (Neelakanta et al, 1997). |
Atelectasis | Use NIV (reintubation rates will be improved- Narita et al, 2010) |
Bleeding | Use a low CVP strategy (2-5mmHg) Intraoperatively, insist on occlusive manoeuvres (eg. Pringle manoeuvre) |
Analgesia | Remember the impaired clearance. Avoid benzodiazepines and long-acting opiates. Epidural seems to increase fluid requirements: use PCA instead. |
Delirium | Hepatic encephalopathy may develop if the patient had abnormal liver function preoperatively. One is referred to the chapter on hepatic encephalopathy for details of its management (spoiler: a lot of lactulose is involved) |
High lactate | Expect it. Unlikely to be related to real shock (more a reflection of poor residual liver function) Observe it. |
Low phosphate | Expect it. Replace it. The phosphate is being absorbed by the regenerating liver. |
Renal failure | In advanced cirrhosis, may represent hepatorenal syndrome. In other situations, it may be pre-renal (i.e. due to inadequate fluid resuscitation) or due to intraoperative renal vascular or ureteric injury. |
Fluid overload | Use concentrated colloids to maintain intravascular volume (eg. 20% albumin). |
Hypercatabolic state | Early enteral nutrition (not much benefit from TPN - Hotta et al, 2002) Optimal pre-operative nutrition is important. Branched-chain amino acids (BCAAs) should be mentioned, in spite of the fact that tey are probably pointless in this setting. |
Hypoglycaemia | Attentive BSL monitoring |
Coagulopathy | Attentive coag monitoring; likely no need for correction unless there is active bleeding |
Infection | Mainly in the setting of bile leaks, abdominal collections, VAP or line-related sepsis. In any case, broad-spectrum cover will be deployed, some combination of an extended-spectrum β-lactam and lactamase inhibitor, eg. Tazocin. |
Jarnagin, William R., et al. "Improvement in perioperative outcome after hepatic resection: analysis of 1,803 consecutive cases over the past decade." Annals of surgery 236.4 (2002): 397-407.
Page, Andrew J., and David A. Kooby. "Perioperative management of hepatic resection." Journal of gastrointestinal oncology 3.1 (2012): 19-27.
Wrighton, Lindsay J., et al. "Postoperative management after hepatic resection." Journal of gastrointestinal oncology 3.1 (2012): 41-47.
Thorat, Ashok, and Wei-Chen Lee. Critical Care Issues After Major Hepatic Surgery. INTECH Open Access Publisher, 2013.
Pagano, Duilio, et al. "The unreliability of continuous postoperative lactate monitoring after extended hepatectomies: single center experience." Updates in surgery 67.1 (2015): 33-37.
Ciuni, Roberto, et al. "Nutritional aspects in patient undergoing liver resection." Updates in surgery 63.4 (2011): 249-252.
Hotta, Tsukasa, et al. "Evaluation of postoperative nutritional state after hepatectomy for hepatocellular carcinoma." Hepato-gastroenterology 50.53 (2002): 1511-1516.
Richter, B., et al. "Nutritional support after open liver resection: a systematic review." Digestive surgery 23.3 (2006): 139-145.
Marchesini, Giulio, et al. "Nutritional supplementation with branched-chain amino acids in advanced cirrhosis: a double-blind, randomized trial." Gastroenterology 124.7 (2003): 1792-1801.
Kim, Say-June, Dong-Goo Kim, and Myung Duk Lee. "Effects of branched-chain amino acid infusions on liver regeneration and plasma amino acid patterns in partially hepatectomized rats." Hepato-gastroenterology 58.109 (2010): 1280-1285.
Neelakanta, Gundappa, et al. "Early tracheal extubation after liver transplantation." Journal of cardiothoracic and vascular anesthesia 11.2 (1997): 165-167.
Narita, Masato, et al. "Noninvasive ventilation improves the outcome of pulmonary complications after liver resection." Internal Medicine 49.15 (2010): 1501-1507.
A 62-year-old man presents to ICU with progressive oliguria and shortness of breath. He had been admitted to the ward a week before because of jaundice. His previous medical history is unremarkable, except for heavy alcohol consumption. There is no history of gastrointestinal bleeding or ingestion of nephrotoxic drugs. There is no past history of renal dysfunction. Clinical
examination reveals a blood pressure of 124/60 mm Hg, jaundice, oedema and a distended non-tender abdomen. Cardiovascular examination is normal.
Investigations reveal:
Ultrasound abdomen: Nodular cirrhosis of liver, ascites and normal sized, regular shaped kidneys.
Urinalysis:
No proteinuria, White cell count <10 X 1~6/L (Normal< 10)
Ascitic tap
White cell count <10 X 10 6 /L (Normal< 10)
Red cell count <10 X 10 6 /L (Normal< 10),
No organisms on Gram stain.
On admission to hospital |
On admission to |
|
Na (135-145 mmol/L) |
139 |
123 |
K (3.5-5.0 mmol/L) |
4.1 |
5.1 |
Creatinine (0.06-0.14 |
0.06 |
0.340 |
Urea (4-6 mmol/L) |
3.8 |
22 |
Bilirubin (0-20 J!mol/L ) |
34 |
48 |
ALT (<40 U/L) |
180 |
600 |
AST(<50 U/L) |
340 |
870 |
Hb (120-1500/L) |
134 |
104 |
wcc (4-11 x 1011/L) |
14.4 |
16.1 |
a) What is the most likely cause of the renal deterioration? Give reasons.
List 4 important management measures specifically for the treatment of this patient's renal dysfunction.
a) What is the most likely cause of the renal deterioration? Give reasons.
Hepatorenal syndrome .
Reasons: Fulfils criteria for Type 1 HRS -(Acute deterioration, absent renal parenchymal disease, absent proteinuria, no shock and no history of nephrotoxic drugs)
List 4 important management measures specifically for the treatment of this patient's renal dysfunction.
Management of complications of renal dysfunction- hyperbilirubinaemia
Albumin administration
Terlipressin I Midodrine/ Octreotide
TIPS
Consideration for liver transplantation
Hepatorenal syndrome is discussed in greater detail in the answer to Question 13 from the first paper of 2001, and in a chapter or rambling digressions.
This case fits the criteria for diagnosis, which are as follows:
i.e. it is renal failure in a patient with coexisting liver failure and no other good reason to have renal failure. "Absence of proteinuria" promotes a view that this is a purely pre-renal disease, with no glomerular damage.
Type 1 hepatorenal syndrome is an acute episode, whereas Type 2 is gradual in onset.
As for "4 important management measures"...
If all else fails, TIPS procedure may be appropriate. As this disorder has an abysmal survival rate, those who qualify for transplant should be worked up for one.
The college examiners prescriptively asked for "management measures specifically for the treatment of this patient's renal dysfunction", but in their answer gave a generic support strategy ("Management of complications of renal dysfunction- hyperbilirubinaemia"). This answer is puzzling on several levels, and probably had no value in terms of marks for the actual trainees, revealing more information about the CICM quality control process than about hepatorenal syndrome.
UpToDate has an excellent synopsis of hepatorenal syndrome.
Salerno, Francesco, et al. "Diagnosis, prevention and treatment of hepatorenal syndrome in cirrhosis." Postgraduate medical journal 84.998 (2008): 662-670.
Solà, Elsa, Mónica Guevara, and Pere Ginès. "Current treatment strategies for hepatorenal syndrome." Clinical Liver Disease 2.3 (2013): 136-139.
Arroyo, Vicente, et al. "Definition and diagnostic criteria of refractory ascites and hepatorenal syndrome in cirrhosis." Hepatology 23.1 (1996): 164-176.
Velez, Juan Carlos Q., and Paul J. Nietert. "Therapeutic response to vasoconstrictors in hepatorenal syndrome parallels increase in mean arterial pressure: a pooled analysis of clinical trials." American Journal of Kidney Diseases 58.6 (2011): 928-938.
Gluud, L. L., M. S. Kjaer, and E. Christensen. "Terlipressin for hepatorenal syndrome." Cochrane Database Syst Rev 4 (2006).
Kalambokis, Georgios, et al. "The effects of chronic treatment with octreotide versus octreotide plus midodrine on systemic hemodynamics and renal hemodynamics and function in nonazotemic cirrhotic patients with ascites."The American journal of gastroenterology 100.4 (2005): 879-885.
Rössle, Martin, and Alexander L. Gerbes. "TIPS for the treatment of refractory ascites, hepatorenal syndrome and hepatic hydrothorax: a critical update." Gut59.7 (2010): 988-1000.
Malinchoc, Michael, et al. "A model to predict poor survival in patients undergoing transjugular intrahepatic portosystemic shunts." Hepatology 31.4 (2000): 864-871.
A patient presented with massive abdominal distension. On examination, a fluid thrill was present. List three clinical diagnoses, which may produce these findings
.
Candidates listing 3 correct differentials for a fluid thrill (see a-c below) or providing 3 conditions which may result in massive ascites (see d-f below) were awarded full marks.
a) Massive ascites
b) Massive ovarian cyst
c) Pregnancy with hydramnios
d) Severe liver disease
e) Budd-Chiari syndrome
f) Severe right heart failure or pericardial constriction
g) Abdominal malignancy.
This question asks for a fairly random detail from Talley and O'Connor.
The "fluid thrill" is one of the tests for ascites; however it will be positive in any condition where a major part of the abdomen is taken up by a big lake of sloshing fluid. Essentially, it consists of somebody (possibly the patient) pressing down on the middle of their abdomen, and the physician percussing one side while feeling for the percussion wave in the other side. A big sloshy lake of fluid will allow the percussion wave to transmit across, whereas bowel gas or stool will transmit nothing.
This generally held to be pretty unreliable sign of ascites, as it requires the ascites to be massive, and under tension.
In either case, the college was fairly non-specific in their expected answer. They would have been happy to hear several different explanations for abdominal fluid (cysts of all sorts, ascites or massive hydramnios are all valid causes). They were equally happy about three different reasons for massive ascites (which leaves the answer pretty well open to any suggestion.. Budd-Chiari? Liver cirrhosis? Severe right heart failure?).
A good description of the fluid thrill test can be found at the website of the University of California, San Diego.
List 4 causes of an elevated serum ammonia concentration in critically ill patients
Hepatic failure
Inherited disorders of urea cycle
Drugs: Valproate, glycine, carbamezapine
Porta-systemic shunts
Increased protein load: GI bleed, TPN,
Infection with urease splitting organisms – proteus Gastric bypass, urinary diversion procedures Cancers – myeloma
Chemotherapy.
This question closely resembles, though is not identical to, Question 14 from the second paper of 2012. There, one may find a discussion of the usefulness of the serum ammonia levels in critical illness.
Here, one is merely expected to regurgitate a series of differentials.
Using a familiar template, an easily remembered list would look like this:
More detail, you beg? Impossibly large tables, useless for the purpose of rapid revision?
Vascular and cardiac causes
Infections
Neoplasms
Drugs
|
Congenital causes
Autoimmune causes
Urinary and renal causes
Endocrine and Metabolic causes
|
Another method of arranging the differentials, according to the physiological mechanism:
Increased substrate for ammoniagenesis
Bypass of normal metabolism
|
Acquired urea cycle defects
Congenital urea cycle defects
Excess of exogenous ammonia
Reabsorption of excreted ammonia
|
Conway, Edward Joseph, and Robert Cooke. "Blood ammonia." Biochemical Journal 33.4 (1939): 457.
Shambaugh, G. E. "Urea biosynthesis I. The urea cycle and relationships to the citric acid cycle." The American journal of clinical nutrition 30.12 (1977): 2083-2087.
McDermott Jr, William V., Raymond D. Adams, and Athol G. Riddell. "Ammonia metabolism in man." Annals of surgery 140.4 (1954): 539.
Vince, Angela, et al. "Ammonia production by intestinal bacteria." Gut 14.3 (1973): 171-177.
Vince, Angela J., and Sigrid M. Burridge. "Ammonia production by intestinal bacteria: the effects of lactose, lactulose and glucose." Journal of medical microbiology 13.2 (1980): 177-191.
Dohrenwend, Paul, and Richard D. Shih. "Glycine Induced Hyperammonemia After Bladder Rupture During Transurethral Resection of a Bladder Tumor." Journal of Medical Cases 4.4 (2013): 250-253.
Felipo, Vicente, and Roger F. Butterworth. "Neurobiology of ammonia." Progress in neurobiology 67.4 (2002): 259-279.
Hashim, Ibrahim A., and Jennifer A. Cuthbert. "Elevated ammonia concentrations: Potential for pre-analytical and analytical contributing factors." Clinical biochemistry 47.16 (2014): 233-236.
Clay, Alison S., and Bryan E. Hainline. "Hyperammonemia in the ICU." CHEST Journal 132.4 (2007): 1368-1378.
Weng, Te-I., Frank Fuh-Yuan Shih, and Wen-Jone Chen. "Unusual causes of hyperammonemia in the ED." The American journal of emergency medicine 22.2 (2004): 105-107.
Hawkes, N. D., et al. "Non-hepatic hyperammonaemia: an important, potentially reversible cause of encephalopathy." Postgraduate medical journal 77.913 (2001): 717-722.
A 54 year old woman was referred to the emergency department by her GP with a 3 day history of vomiting accompanied by upper abdominal pain. On examination she was obese, appeared restless and confused, GCS 13, febrile 38.6 C, heart rate of 100 /min, BP 90/40 mm Hg. She has Spo2 of 88% on oxygen via a non-rebreather bag. There was diffuse abdominal tenderness on palpation in particular in the upper abdomen. Bowel sounds were sluggish. Blood tests taken in a private laboratory the preceding day had revealed a lipase of 400 U/l (normal < 70).
23.1) What are the differential diagnoses of this patient’s presentation?
23.2) What are the causes of hypotension in acute pancreatitis?
23.3) List 3 causes of a raised A-a gradient in acute pancreatitis?
23.4) What do you understand is the role for prophylactic antimicrobial therapy in sterile pancreatic necrosis?
23.1) What are the differential diagnoses of this patient’s presentation?
1) Pancreatitis
2) Perf DU
3) Intestinal obstruction
4) Acute cholecystitis with sepsis
5) Aspiration and sepsis
6) Gut ischaemia
23.2) What are the causes of hypotension in acute pancreatitis?
a) sequestration (3rd spacing) of protein rich fluids in and around the pancreas and abbdominal cavity, retroperitoneum
b) compounded by pre existing fluid depletion.c) direct myocardial depression
d) SIRS / sepsis
e) Intra-abd hypertensionf) Bleeding
23.3) List 3 causes of a raised A-a gradient in acute pancreatitis?
Pulmonary dysfunction - Aspiration, pleural effusions, ARDS, atelectasis.
23.4) What do you understand is the role for prophylactic antimicrobial therapy in sterile pancreatic necrosis?
a) Antibiotic use in SAP without overt infection controversial and trial data are conflicting.
b) Antibiotics have been given either IV or IV plus orally/rectally via SDD.
c) Early trials - underpowered, mostly non blinded and included patients with differing disease severity suggested a reduction in both infections and improved outcome with early use of prophylactic antibiotics (Cefuroxime and imipenem) in necrotising SAP when compared with placebo. Subsequent meta analyses including a Cochrane review also suggested that antibiotics reduced infections and mortality and need for surgery in necrotic pancreatitis.
d) 2 recent RCTs (Isenmann 2004 and Dellinger 2007) have however demonstrated no effect on outcome or infection rate when prophylactic antibiotics were used in necrotic pancreatitis. The SCCM (2004) consensus conference on severe pancreatitis recommends against the use of routine prophylactic antibiotics.
23.1) What are the differential diagnoses of this patient’s presentation?
An obese hypoxic woman presents in a state of shock, febrile, and with this story of vomiting and abdominal pain. A systematic approach is called for, even though the lipase is high and the obvious single diagnosis is pancreatitis.
Vascular causes:
Infectious causes: sepsis from any origin, but more likely the gut,
Neoplastic causes
Drug-induced causes
Autoimmune causes eg. inflammatory bowel disease with perforation
Traumatic causes eg. Boerhaave's syndrome due to excessive vomiting
Endocrinological cause of abdo pain and SIRS, eg. pacreatitis due to any number of causes
23.2) What are the causes of hypotension in acute pancreatitis?
Again, there are several mechanisms:
23.3) List 3 causes of a raised A-a gradient in acute pancreatitis?
23.4) What do you understand is the role for prophylactic antimicrobial therapy in sterile pancreatic necrosis?
Again, thank you to Rajkumar (you know who you are) for pointing out the error in this answer. There was a duplication of content from Question 22 of the same paper. Well spotted!
Pederzoli, Paolo, et al. "A randomized multicenter clinical trial of antibiotic prophylaxis of septic complications in acute necrotizing pancreatitis with imipenem." Surgery, gynecology & obstetrics 176.5 (1993): 480-483.
Wilmer, Alexander. "ICU management of severe acute pancreatitis." European journal of internal medicine 15.5 (2004): 274-280.
Villatoro, Eduardo, Mubashir Mulla, and Mike Larvin. "Antibiotic therapy for prophylaxis against infection of pancreatic necrosis in acute pancreatitis."Cochrane Database Syst Rev 5.5 (2010).
List 4 clinical signs of portal hypertension.
1. Splenomegaly
2. Ascites
3. Caput medusae
4. Haemorrhoids
This question is identical to Question 5.1 from the second paper of 2010.
List 4 clinical signs of portal hypertension.
List 4 clinical signs of portal hypertension.
• Splenomegaly
• Caput medusae
• Ascites
• Haemorrhoids on rectal examination
• Haematemesis? Melaena
This is fairly straightforward.
Escorsell, Angels, Joan Carles García-Pagán, and Jaume Bosch. "Assessment of portal hypertension in humans." Clinics in liver disease 5.3 (2001): 575-589.
de Franchis, Roberto. "Revising consensus in portal hypertension: report of the Baveno V consensus workshop on methodology of diagnosis and therapy in portal hypertension." Journal of hepatology 53.4 (2010): 762-768.
A 26 year old female presents to the Emergency Department having been found at home confused and jaundiced by her GP.
Her GCS is E3V5M5
She has a temperature of 38 ˚C, BP 90/60, HR 90 and SpO2 94% on 4 litres/min O2.
Her plasma biochemistry is as follows:
Test |
Value |
Normal Range |
Sodium |
137 mmol/L |
(135 – 145) |
Potassium |
4.1 mmol/L |
(3.5 – 5.0) |
Total bilirubin* |
200 micromol/L |
(0 – 25) |
AST* |
4000 U/L |
(<40) |
GGT* |
500 U/L |
(<40) |
ALT* |
3000 U/L |
(<40) |
Urea |
4.2 mmol/L |
(4 – 6) |
Coagulation profile:
Test |
Value |
Normal Range |
INR* |
2.6 |
(0.8 – 1.2) |
a) Based on the above information, what do you think this woman is suffering from?
b) List 5 important aetiologies which could result in this presentation.
c) List 4 important complications (apart from respiratory failure) she is at risk of developing.
d) List 4 reasons why this woman might progress to developing respiratory failure.
a) Based on the above information, what do you think this woman is suffering from?
Acute liver failure (without more detail it is hard to say, hyperacute, acute, subacute or chronic)
b) List 5 important aetiologies which could result in this presentation.
• Sepsis
• Viral hepatitis – Hep B/C/D, CMV/EBV
• Drug induced
• Poisoning
• Miscellaneous (wilson’s disease, acute fatty liver of pregnancy, ischaemic necrosis, Budd-Chiari, complications of hepatic surgery)
• Idiopathic
• Pregnancy related
c) List 4 important complications (apart from respiratory failure) she is at risk of developing.
• Cerebral oedema and herniation
• Coagulopathy
• GI bleed
• Sepsis
• Renal failure
d) List 4 reasons why this woman might progress to developing respiratory failure.
• Impaired ventilation because of coma
• Pleural effusions
• ARDS
• Intra-pulmonary shunts
• Aspiration pneumonia
• Sepsis- pulmonary or extrapulmonary
a) Based on the above information, what do you think this woman is suffering from?
With the transaminases in their thousands, a raised bilirubin, coagulopathy and fever, one might assume that this lady has some sort of acute hepatitis. As the college rightly points out, "acute hepatic failure" is the only description one can make without any further history.
b) List 5 important aetiologies which could result in this presentation.
c) List 4 important complications (apart from respiratory failure) she is at risk of developing.
There is a brilliant article on this subject which outlines all the possible and impossible complications of acute hepatic failure. I will summarise their Panel 1:
Complications of acute liver failure
d) List 4 reasons why this woman might progress to developing respiratory failure.
This answer calls for differentials of respiratory failure in a semiconscious shocked woman with fever.
Bernal, William, et al. "Acute liver failure." The Lancet 376.9736 (2010): 190-201.
Fallon, Michael B., and Gary A. Abrams. "Pulmonary dysfunction in chronic liver disease." Hepatology 32.4 (2000): 859-865.
Answer the following questions about transjugular intrahepatic portosystemic shunts (TIPS):
a) What is a TIPS procedure and why is it used in patients with portal hypertension?
b) What are 2 recognised indications for this procedure?
c) Excluding mortality list 5 COMMON complications of TIPS procedure
d) Describe one classification system used in assessing severity of chronic liver disease and outline its utility.
a) What is a TIPS procedure and why is it used in patients with portal hypertension?
The hepatic vein is accessed via the internal jugular vein and IVC. A needle is then passed to connect the hepatic vein with the large portal vein near the centre of the liver, the needle tract dilated and a stent inserted to maintain the tract and form the shunt between the higher pressure portal vein and the lower pressure hepatic vein. This reduces portal hypertension.
b) What are 2 recognised indications for this procedure?
a) Variceal bleeding that has failed endoscopic and pharmacological treatment. b) Refractory ascites
c) Excluding mortality list 5 COMMON complications of TIPS procedure
• thrombosis
• occlusion of the stent
• capsular puncture
• bleeding
• encephalopathy
• stent migration
d) Describe one classification system used in assessing severity of chronic liver disease and outline its utility.
Either: Childs-Pugh score
Classified A,B or C by a composite of Total bilirubin, albumin, INR, ascites and hepatic encephalopathy. Originally used for prognostication for surgery – also used for prognostication in chronic liver disease and prediction of likelihood of complications of cirrhosis
Or: MELD score severity scoring system for assessing severity of chronic liver that uses the serum bilirubin, creatinine and INR. Initially developed to predict three month survival in patients post TIPS. Now used for prognosis of liver disease and prioritizing liver transplant recipients
The first 3 parts of this question are reasonably straightforward.
Indications for TIPS:
Technical complications of TIPS:
Complications from portal venous shunting:
The last part of the question is far from straightforward. How does one "briefly outline the utility" of something like the Childs-Pugh scoring system?
The system itself has existed since the 1970s, and it includes in its assessment the albumin level, the prothrombin time, the bilirubin, the presence of encephalopathy and the presence of ascites.
It seems the main point is prognosis. The utility of the scoring systems in general has been to estimate the expected lifespan of a cirrhosis patient, which has implications for liver transplantation. However, in the ICU, the overall mortality for cirrhosis patients is about 43%, irrespective of Childs-Pugh score. In general, SOFA and APACHE are better systems for predicting in-ICU mortality than the liver-specific scoring systems.
Thus, the answer to the "briefly outline utility" question should look like this:
Classification system:
Its utility, briefly:
Ochs, Andreas, et al.New England Journal of Medicine 332.18 (1995): 1192-1197."The transjugular intrahepatic portosystemic stent–shunt procedure for refractory ascites."
Guevara, Mónica, et al. "Transjugular intrahepatic portosystemic shunt in hepatorenal syndrome: effects on renal function and vasoactive systems."Hepatology 28.2 (1998): 416-422.
Papatheodoridis, George V., et al. "Transjugular intrahepatic portosystemic shunt compared with endoscopic treatment for prevention of variceal rebleeding: A meta‐analysis." Hepatology 30.3 (1999): 612-622.
Azoulay, D., et al. "Transjugular intrahepatic portosystemic shunt (TIPS) for severe veno-occlusive disease of the liver following bone marrow transplantation." Bone marrow transplantation 25.9 (2000).
Ganger, Daniel R., et al. "Transjugular intrahepatic portosystemic shunt (TIPS) for Budd-Chiari syndrome or portal vein thrombosis." The American journal of gastroenterology 94.3 (1999): 603-608.
Freedman, A. M., et al. "Complications of transjugular intrahepatic portosystemic shunt: a comprehensive review." Radiographics 13.6 (1993): 1185-1210.
Angermayr, B., et al. "Child-Pugh versus MELD score in predicting survival in patients undergoing transjugular intrahepatic portosystemic shunt." Gut 52.6 (2003): 879-885.
Bazarah, Salem M., et al. "Utility of MELD and Child-Turcotte-Pugh scores and the Canadian waitlisting algorithm in predicting short-term survival after liver transplant." Clinical and investigative medicine 27 (2004): 162-167.
Singh, Nina, et al. "Outcome of patients with cirrhosis requiring intensive care unit support: prospective assessment of predictors of mortality." Journal of gastroenterology 33.1 (1998): 73-79.
Levesque, Eric, et al. "Prospective evaluation of the prognostic scores for cirrhotic patients admitted to an intensive care unit." Journal of hepatology 56.1 (2012): 95-102.
These are the biochemical results taken from a 48-year-old man, missing from an alcohol rehabilitation program and found in his home comatose by police three days from the time he was last seen.
Parameter |
Patient Value |
Normal Range |
||||
Sodium |
126* mmol/l |
138 – 145 |
||||
Potassium |
3.5 mmol/l |
3.5 – 5.2 |
||||
Creatinine |
250*µmol/l |
40– 100 |
||||
Urea |
7.0 mmol/l |
3.1– 7.5 |
||||
Bilirubin (total) |
509*µmol/l |
2–22 |
||||
Protein (total) |
40* g/l |
65– 85 |
||||
Albumin |
20* g/l |
38– 48 |
||||
ALP |
153* IU/l |
40– 100 |
||||
GGT |
459* IU/l |
0–50 |
||||
ALT |
336* IU/l |
0 - 45 |
||||
CK |
400* IU/l |
30– 180 |
||||
Troponin |
0.1 µg/l |
0–0.3 |
||||
Glucose |
3.2* mmol/l |
3.5– 4.6 |
||||
Ammonia |
342*µmol/l |
0–50 |
||||
Lactate |
3.7* mmol/l |
0.6– 2.4 |
a) Given this presentation, list 3 possible causes of his altered conscious state?
b) Interpret the biochemical abnormalities
a) Given this presentation, list 3 possible causes of his altered conscious state?
Alcohol intoxication
Hepatic encephalopathy
Drug ingestion
GI bleed
Sepsis
Intracranial bleed
b) Interpret the biochemical abnormalities
Liver dysfunction as demonstrated by elevated enzymes, reduced albumin, hypoglycaemia with decompensation indicated by marked elevation of ammonia. Hyponatraemia in keeping with cirrhosis. Raised lactate as a result of liver dysfunction / alcoholic ketoacidosis / sepsis / thiamine deficiency Raised creatinine indicates renal dysfunction and urea may be apparently “normal” because of decreased hepatic dysfunction and possible nutritional deficiencies. Urea:creatine ratio suggests that GI bleed and/or dehydration are unlikely
a)
Ok, the college has given us an alcoholic and put 342 µmol/l of ammonia into him.
Thus one of the differentials would have to be hepatic encephalopathy.
Given the extent of the other liver function abnormalities, and of course the borderline BSL, one could surmise that hypoglycaemia is another valid differential.
Being a drunk, alcohol intoxication is not out of the question.
One might wish to use the VINDICATE acronym to come up with some differentials.
Fortunately, the college only asks for three answers. Hepatic encephalopathy, alcohol intoxication and sepsis seem the most likely, given the bloods.
b) Interpret the biochemical abnormalities
There is some good material out there on the biochemical abnormalities of chronic liver disease, and their interpretation.
Chung, Raymond T., David L. Jaffe, and Lawrence S. Friedman. "Complications of chronic liver disease." Critical care clinics 11.2 (1995): 431-463.
Heidelbaugh, Joel J., and Michael Bruderly. "Cirrhosis and chronic liver failure: part II. Complications and treatment." American family physician 74.5 (2006).
A 40-year-old woman presents 7 days after a pan-colectomy for Crohn’s disease. She has a past history of antithrombin III deficiency. She has increasing abdominal pain and vomiting. There is marked tenderness in the right upper quadrant. An abdominal CT scan is performed.
a) CT Scan:
b) Cause:
Portal venous thrombosis
c) Causes of anti-thrombin III deficiency:
Hereditary
Acquired
o Post-operative state
o Liver disease
o Disseminated intravascular coagulation
o Nephrotic syndrome
o Vasculitis
e) Treatment:
The CT scan I used was mined shamelessly from Google Images. It is not the one which appeared in the paper.
The first two parts of this question are straightforward.
The CT features of portal vein thrombosis are as follows:
The second half of the question requires detailed knowledge of AT-III deficiency. One can arrive at at least half of a sensible answer by logically asking why AT-III might not be present in sufficient quantities. Either you hereditarily fail to synthesise enough of it, or your liver is so damaged that it cannot produce enough. Or, it has been used up somehow, eg. in the context of DIC, MAHA, or in a bypass circuit. Lastly, it is possible that you are losing it along with other proteins via your leaky nephrotic kidneys.
The chapter which discusses thrombophilia screening tests contains within it this table, which lists the causes of AT-III deficiency with greater granularity.
Inherited AT-III defects
|
Acquired Reduced production
Loss of protein
Increased consumption
|
The management of AT-III deficiency is, predictably, supplementation with AT-III. If the expensive purified factor is not available, FFP will suffice. Heparinisation for thrombosis actually will not work unless there is some antithrombin for the heparin to act upon.
Management options for PVT include the following:
UpToDate offers a good article about Antithrombin III deficiency, for a price.
Beresford, C. H. "Antithrombin III deficiency." Blood reviews 2.4 (1988): 239-250.
Mathieu, Didier, Norbert Vasile, and P. Grenier. "Portal thrombosis: dynamic CT features and course." Radiology 154.3 (1985): 737-741.
Lee, Hae-Kyung, et al. "Portal vein thrombosis: CT features." Abdominal imaging 33.1 (2008): 72-79.
Denninger, Marie‐Hélène, et al. "Cause of portal or hepatic venous thrombosis in adults: the role of multiple concurrent factors." Hepatology31.3 (2000): 587-591.\
Valla, Dominique-Charles, and Bertrand Condat. "Portal vein thrombosis in adults: pathophysiology, pathogenesis and management." Journal of hepatology 32.5 (2000): 865-871.
Boyer, Thomas D. "Management of portal vein thrombosis."Gastroenterology & hepatology 4.10 (2008): 699.
Basit, Syed Abdul, Christian D. Stone, and Robert Gish. "Portal vein thrombosis." Clinics in liver disease 19.1 (2015): 199-221.
A 42-year-old male is admitted to ICU following a cadaveric orthotopic liver transplant for end-stage liver disease secondary to alcohol-induced cirrhosis.
a)
Haemodynamic stabilization – optimize cardiac output and tissue perfusion and avoid fluid overload as ventricular function may be impaired. Close haemodynamic monitoring. Vaso-active agents as indicated.
Correction of anaemia and coagulopathy – maintain haemocrit 0.25 – 0.3 to keep blood viscosity low. INR £2, APTT £50 secs, Fibrinogen above 0.5 g/L and Platelets above 30 x 109/L.
Fluid and electrolyte management – appropriate negative fluid balance day 1 decreases risk of pulmonary complications. Fluid overload may aggravate graft congestion and oedema caused by ischaemic-reperfusion. Electrolyte imbalances are common and need to be corrected.
Correction of metabolic abnormalities – hypoglycaemia is an ominous sign of compromised liver recovery, hyperglycaemia also may occur, acid-base abnormalities also occur
Early weaning from mechanical ventilation – associated with better outcome but not feasible in patients with respiratory failure, haemodynamic instability, pulmonary oedema, primary graft dysfunction, encephalopathy etc. Unsucessful early extubation may result in impaired oxygen delivery to transplanted liver
Monitoring of graft function LFTs, lactate, BSL, coagulation, hepatic artery doppler
Early detection of surgical complications - bleeding
Immunosuppressants
Infection prophylaxis
Housekeeping including analgesia (PCA) and appropriate nutrition plan
Other – ICP monitoring if decompensated CLD pre-op
b)
Delayed metabolism of sedative / anaesthetic drugs
Metabolic derangements – hypoglycaemia, hyponatraemia, hyperosmolar syndrome
Hepatic encephalopathy
Hypoxic-ischaemic cerebral injury
Seizures
Intracerebral haemorrhage
This is a delicate topic for anybody who has not had the privilege of working in one of the sandstone centres of excellence, where liver transplants for the rich and famous serve as a constant source of private funding and media attention.
Thus, let there be a brief digression into the specifics of managing a post-op orthotopic liver transplant patient in the ICU.
(Incidentally, for those like me who were unsure precisely what the long words mean, "orthotopic" transplantation is the total removal of the old diseased liver, and the implantation of the donated organ into the same anatomical position).
The above link points to a scanned document from the Atlas of Organ Transplantation, which deals extensively with the pre-operative and peri-operative management of liver transplantation, detailing such interesting time intervals as "the Anhepatic stage". It however, is more aimed at the surgeons. Because the quality of the scanner leaves much to be desired, the gory pictures of open livers are just black-and-white blurs.
However, there is plenty of literature regarding the critical care management of liver transplant recipients.
Let us approach this answer systematically.
The second part of this question is far less interesting, and refers mainly to the candidate's ability to generate differentials for a decreased level of consciousness.
Using a usual template, one can arrive at a series of differential easily:
The best, most comprehensive source is this article:
Feltracco, Paolo, et al. "Intensive care management of liver transplanted patients." World journal of hepatology 3.3 (2011): 61.
The rest are also helpful.
Stieber, Andrei C., R. D. Gordon, and J. R. Galloway. "Orthotopic liver transplantation." Hepatology. A text book of liver disease. WB Saunders, Philadelphia (1996): 1759-1780.
Mazariegos, George V., Ernesto P. Molmenti, and David J. Kramer. "Early complications after orthotopic liver transplantation." Surgical Clinics of North America 79.1 (1999): 109-129.
Razonable, Raymund R., et al. "Critical care issues in patients after liver transplantation." Liver Transplantation 17.5 (2011): 511.
Mandell, M. Susan, et al. "Reduced use of intensive care after liver transplantation: influence of early extubation." Liver Transplantation 8.8 (2002): 676-681.
Critically evaluate the role of the following investigations in the critically ill patient:
This question was quite arbitrarily placed into the gastroenterology section, on no grounds other than the interesting association of ammonia with various metabolic disturbances, versus the rather boring association of BNP with heart failure.
The question asks us to "critically evaluate", which means a certain systematic approach. I note that this approach was not used in the college answer.
The use of ammonia levels in critical care:
The use of BNP levels in critical care:
Clay, Alison S., and Bryan E. Hainline. "Hyperammonemia in the ICU." CHEST Journal 132.4 (2007): 1368-1378.
Ong, Janus P., et al. "Correlation between ammonia levels and the severity of hepatic encephalopathy." The American journal of medicine 114.3 (2003): 188-193.
McKinney, A. M., et al. "Acute hepatic (or hyperammonemic) encephalopathy: diffuse cortical injury and the significance of ammonia." American Journal of Neuroradiology 32.7 (2011): E142-E142.
Richter, Derek, and Rex MC Dawson. "The ammonia and glutamine content of the brain." Journal of Biological Chemistry 176.3 (1948): 1199-1210.
Summar, Marshall L., et al. "Unmasked adult-onset urea cycle disorders in the critical care setting." Critical care clinics 21.4 (2005): S1-S8.
Troughton, Richard W., et al. "Treatment of heart failure guided by plasma aminoterminal brain natriuretic peptide (N-BNP) concentrations." The Lancet355.9210 (2000): 1126-1130.
McCullough, Peter A., et al. "B-type natriuretic peptide and clinical judgment in emergency diagnosis of heart failure analysis from Breathing Not Properly (BNP) multinational study." Circulation 106.4 (2002): 416-422.
Hall, Christian. "Essential biochemistry and physiology of (NT-pro) BNP."European Journal of Heart Failure 6.3 (2004): 257-260.
Silver, Marc A., et al. "BNP Consensus Panel 2004: A clinical approach for the diagnostic, prognostic, screening, treatment monitoring, and therapeutic roles of natriuretic peptides in cardiovascular diseases." Congestive Heart Failure 10.s5 (2004): 1-30.
Jefic, Dane, et al. "Utility of B-type natriuretic peptide and N-terminal pro B-type natriuretic peptide in evaluation of respiratory failure in critically ill patients."CHEST Journal 128.1 (2005): 288-295.
Karmpaliotis, Dimitri, et al. "Diagnostic and prognostic utility of brain natriuretic Peptide in subjects admitted to the ICU with hypoxic respiratory failure due to noncardiogenic and cardiogenic pulmonary edema." CHEST Journal 131.4 (2007): 964-971.
Demir, Ali, et al. "The value of Serum BNP for diagnosis of intracranial injury in minor head trauma." World journal of emergency surgery 9.1 (2014): 16.
Li, Nan, et al. "BNP and NT-proBNP levels in patients with sepsis." Ront Biosei18 (2013): 1237-1243.
Mitaka, Chieko, et al. "Increased plasma concentrations of brain natriuretic peptide in patients with acute lung injury." Journal of critical care 12.2 (1997): 66-71.
Kucher, Nils, Gert Printzen, and Samuel Z. Goldhaber. "Prognostic role of brain natriuretic peptide in acute pulmonary embolism." Circulation 107.20 (2003): 2545-2547.
Meyer, Brigitte, et al. "511: Nt-Pro-Bnp Is A Strong Predictor of Outcome in Critically Ill Patients Admitted to the Icu After Cardiac Surgery." Critical Care Medicine 39.12 (2011): 141.
Yardan, T., et al. "B-type natriuretic peptide as an indicator of right ventricular dysfunction in acute pulmonary embolism." International journal of clinical practice 62.8 (2008): 1177-1182.
These are the biochemical results taken from a 48-year-old male, missing from an alcohol rehabilitation program and found in his home, comatose, by police, three days from the time he was last seen.
Parameter | Patient Value | Normal Adult Range |
Sodium | 126 mmol/L* | 138 – 145 |
Potassium | 3.5 mmol/L | 3.5 – 5.0 |
Creatinine | 250 μmol/L* | 40 – 100 |
Urea | 7.0 mmol/L | 3.1 – 7.5 |
Bilirubin (total) | 509 μmol/L * | 2.0 – 22.0 |
Protein (total) | 40 g/L* | 65 – 85 |
Albumin | 20 g/L* | 38 – 48 |
Alkaline phosphatase (ALP) | 153 IU/L* | 40 – 100 |
Gamma glutamyl transferase (GGT) | 459 IU/L* | 0 – 50 |
Alanine aminotransferase (ALT) | 336 IU/L* | 0 – 45 |
Creatine kinase (CK) | 400 IU/L* | 30 – 180 |
Glucose | 3.2 mmol/L* | 3.5 – 4.6 |
Ammonia | 342 μmol/L | 0 – 50 |
Lactate | 3.7 mmol/L* | 0.6 – 2.4 |
a) Comment, with explanation, on each of the biochemical abnormalities.
b) List three possible causes of his altered conscious state.
a) Comment, with explanation, on each of the biochemical abnormalities.
Liver dysfunction as demonstrated by elevated enzymes, reduced albumin. Hypoglycaemia with decompensation indicated by marked elevation of ammonia. Hyponatraemia in keeping with cirrhosis.
Raised lactate as a result of liver dysfunction / alcoholic ketoacidosis / sepsis / thiamine deficiency
Raised creatinine indicates renal dysfunction and urea may be apparently “normal” because of
decreased hepatic dysfunction and possible nutritional deficiencies. Urea:creatinine ratio suggests
that GI bleed and/or dehydration are unlikely
b) List three possible causes of his altered conscious state.
This question is nearly identical to Question 29.2 from the second paper of 2011. The answer to that question is also duplicated here, to simplify revision
The following is a list of biochemical abnormalities and plausible explanations for them:
The college asks for three possible causes of unconsciousness in this comatose drunk. Three only. The most relevant ones would have to be
A possible list of differentials would have to also include the following:
Johnston, David E. "Special considerations in interpreting liver function tests." American family physician 59 (1999): 2223-2232.
Limdi, J. K., and G. M. Hyde. "Evaluation of abnormal liver function tests." Postgraduate medical journal 79.932 (2003): 307-312.
Hoekstra, Lisette T., et al. "Physiological and biochemical basis of clinical liver function tests: a review." Annals of surgery 257.1 (2013): 27-36.
Giannini, Edoardo G., Roberto Testa, and Vincenzo Savarino. "Liver enzyme alteration: a guide for clinicians." Canadian medical association journal 172.3 (2005): 367-379.
Kim, W., et al. "Serum activity of alanine aminotransferase (ALT) as an indicator of health and disease." Hepatology 47.4 (2008): 1363-1370.
Pratt, Daniel S., and Marshall M. Kaplan. "Evaluation of abnormal liver-enzyme results in asymptomatic patients." New England Journal of Medicine 342.17 (2000): 1266-1271.
Whitfield, J. B. "Gamma glutamyl transferase." Critical reviews in clinical laboratory sciences 38.4 (2001): 263-355.
Balistreri, William F., et al. "Intrahepatic cholestasis: Summary of an American Association for the Study of Liver Diseases single‐topic conference." Hepatology 42.1 (2005): 222-235.
Assy, N., et al. "Diagnostic approach to patients with cholestatic jaundice." World journal of gastroenterology 5.3 (1999): 252-262.
Stellpflug, Samuel J. "Transaminitis: The Lab Test That Has Inflammation…." Journal of Medical Toxicology 7.3 (2011): 252-253.
A 43-year-old female with a history of paranoid schizophrenia and multiple episodes of selfharm, presented to the Emergency Department with decreased conscious state. She had been seen three days earlier for possible worsening of her psychosis and discharged home.
Her haematology and biochemistry results at both presentations are as follows:
Parameter | Patient Value | Normal Adult Range | |
1st Presentation | 2nd Presentation | ||
Haemoglobin | 134 g/L | 135 g/L | 115 – 160 |
White cell count | 12.6 x 109 /L* | 7.5 x 109/L | 4.0 – 11.0 |
Platelet count | 250 x 1012/L | 76 x 1012/L* | 150 – 400 |
Prothrombin time | 40.0 seconds* | 12.3 – 16.6 | |
International Normalised Ratio (INR) | 4.1* | 0.9 – 1.3 | |
Activated partial thromboplastin time (APTT) | 35.0 seconds | 27.0 – 38.5 | |
Fibrinogen | 1.8 g/L* | 2.0 – 4.0 | |
Sodium | 139 mmol/L | 136 mmol/L | 134 – 146 |
Potassium | 3.6 mmol/L | 4.6 mmol/L | 3.4 – 5.0 |
Bicarbonate | 18 mmol/L* | 21 mmol/L* | 22 – 32 |
Urea | 5.0 mmol/L | 18.9 mmol/L* | 3.0 – 8.0 |
Creatinine | 56 µmol/L | 448 µmol/L* | 45 – 90 |
Bilirubin total | 6.0 µmol/L | 81 µmol/L* | < 20 |
Alanine aminotransferase (ALT) | 31 U/L | 11700 U/L* | < 35 |
Alkaline phosphatase (ALP) | 88 U/L | 245 U/L* | 35 – 135 |
Gamma glutamyl transferase (GGT) | 13 U/L | 104 U/L* | < 40 |
Lactate | 4 mmol/L | < 1.5 | |
Paracetamol | < 10 mg/L | 10 mg/L | |
Urine Ethanol | Not detected | ||
Urine Amphetamines | Not detected | ||
Urine Benzodiazepines | Detected | ||
Urine Cannabinoids | Not detected | ||
Urine Opiates | Not detected |
a) What is the underlying diagnosis?
b) List the possible causes in this patient.
c) Outline your immediate management.
a) What is the underlying diagnosis?
Hyperacute (fulminant) hepatic failure
b) List the possible causes in this patient.
Paracetamol
Idiosyncratic drug reaction
Other toxin e.g. Amanita phalloides
Viral (hepatitis A and E as hyperacute consider B, CMV, Epstein Barr)
Ischaemic hepatitis
Budd-Chiari
c) Outline your immediate management.
Concurrent resuscitation and institution of supportive care and monitoring with focussed assessment
to identify underlying cause and definitive management as indicated.
Airway management and oxygen – likely to need intubation and mechanical ventilation with
ARDSNet targets and PCO2 32-38 mmHg
Haemodynamic support for adequate MAP and CPP >60 and appropriate monitoring (A-line,
PAC/PiCCO, bedside echo etc.). Avoid fluid overload
Strategies to offset cerebral oedema (head-up, neutral position, sedation, PCO2 targets, Na 145-155
etc.)
ICP monitoring controversial
Consider renal replacement therapy
Extracorporeal albumin dialysis therapies (SPAD, MARS and Prometheus) have limited evidence to
support use
Monitoring of coagulopathy using TEG/ROTEM to guide correction. Correction to cover invasive
procedures or if bleeding otherwise not.
Monitor blood glucose
Screening for infection and antibiotics as indicated +/- empiric broad spectrum cover including antifungal
Stress ulcer prophylaxis
Nutrition
Specific
NAC
Lactulose / Neomycin / Rifaximin controversial and use varies from unit to unit
Investigations including liver USS, viral screen
Discussion with liver transplant team / transfer to ICU with liver unit (does not currently meet
transplantation criteria)
The diagnosis is clearly liver failure. The college further qualify their description as "fulminant". That term is very 1990s. What makes a liver failure fulminate? Well, apparently that just means it is acute (see Sass et al, 2005). The term was originally applied to patients in whom encephalopathy developed within two weeks of the development of jaundice. These days, the terminology we use is hyparcute (0-7 days), acute (8-28 days) and subacute (29 days to 8 weeks). The disorders were redefined by Williams et al in 1993, on the basis of the fact that they correlate better with survival statistics (paradoxically, the hyperacute ones do much better).
Possible categories of the causes:
Vascular:
Infectious:
Neoplastic:
Drug-induced:
|
Idiopathic: idiosyncratic drug reactions:
Congenital:
Autoimmune:
Traumatic:
Endocrine/metabolic:
|
Of these, the following short list is relevant. However, it must be added that the college did not specify how many differentials they wanted. You could have brought up your various azathiprine-induced hepatitis and Amanita phalloides mushrooms.
Thus:
Management
As with most things, management falls into the category of specific management and supportive management. Specific management may be viewed as a series of antidotes or proven solutions tailored to specific causes of the liver failure.
Paracetamol overdose |
N-acetylcystine |
Wilson's disease (chronic) |
Copper chelating agents |
Hepatic vein thrombosis |
Thrombolysis / clot retrieval; TIPS procedure |
Hepatitis viruses |
Antiviral drugs |
Autoimmune hepatitis |
Steroids may be helpful... or may be harmful. |
Alcoholic hepatitis |
Steroids are probably helpful |
Valproate overdose |
L-carnitine, and probably also dialysis to extract the excess ammonia |
Amanita phalloides |
Haemopherfusion may remove the phallotoxins |
A discussion of supportive management might take the following shape:
Sass, David A., and A. Obaid Shakil. "Fulminant hepatic failure." Liver Transplantation 11.6 (2005): 594-605.
Williams, R., S. W. Schalm, and J. G. O'Grady. "Acute liver failure: redefining the syndromes." The Lancet 342.8866 (1993): 273-275.
Chapter 44 (pp. 501) Liver failure by Christopher Willars and Julia Wendon
Daly, Frank FS, et al. "Guidelines for the management of paracetamol poisoning in Australia and New Zealand-explanation and elaboration." Medical journal of Australia 188.5 (2008): 296.
Bailey, Benoit, René Blais, and Anne Letarte. "Status epilepticus after a massive intravenous N-acetylcysteine overdose leading to intracranial hypertension and death." Annals of emergency medicine 44.4 (2004): 401-406.
Parsons-Smith, B. G., et al. "The electroencephalograph in liver disease." The Lancet 270.7001 (1957): 867-871.
Ramos, Juan Francisco Rivera, and Celina Rodríguez Leal. "Review of the final report of the 1998 Working Party on definition, nomenclature and diagnosis of hepatic encephalopathy." Ann Hepatol 10 (2011): S36-S39.
Walsh, Timothy S., et al. "Energy expenditure in acetaminophen-induced fulminant hepatic failure." Critical care medicine 28.3 (2000): 649-654.
Ichai, Philippe, et al. "Usefulness of corticosteroids for the treatment of severe and fulminant forms of autoimmune hepatitis." Liver transplantation 13.7 (2007): 996-1003.
O’Grady, John G., et al. "Early indicators of prognosis in fulminant hepatic failure." Gastroenterology 97.2 (1989): 439-445.
Dhiman, Radha K., et al. "Early indicators of prognosis in fulminant hepatic failure: An assessment of the Model for End‐Stage Liver Disease (MELD) and King's College Hospital Criteria." Liver transplantation 13.6 (2007): 814-821.
Yantorno, Silvina E., et al. "MELD is superior to King's college and Clichy's criteria to assess prognosis in fulminant hepatic failure." Liver transplantation13.6 (2007): 822-828.
Wiesner, Russell, et al. "Model for end-stage liver disease (MELD) and allocation of donor livers." Gastroenterology 124.1 (2003): 91-96.
Gleisner, Ana L., et al. "Survival benefit of liver transplantation and the effect of underlying liver disease." Surgery 147.3 (2010): 392-404.
Ding, G. K. A., and N. A. Buckley. "Evidence and consequences of spectrum bias in studies of criteria for liver transplant in paracetamol hepatotoxicity." QJM101.9 (2008): 723-729.
Lee, William M., R. Todd Stravitz, and Anne M. Larson. "Introduction to the revised American Association for the Study of Liver Diseases Position Paper on acute liver failure 2011." Hepatology 55.3 (2012): 965-967.
McPhail, Mark JW, Julia A. Wendon, and William Bernal. "Meta-analysis of performance of Kings’s College Hospital Criteria in prediction of outcome in non-paracetamol-induced acute liver failure." Journal of hepatology 53.3 (2010): 492-499.
Stravitz, R. Todd, et al. "Intensive care of patients with acute liver failure: recommendations of the US Acute Liver Failure Study Group." Critical care medicine 35.11 (2007): 2498-2508.
Warrillow, S. J., and R. Bellomo. "Preventing cerebral oedema in acute liver failure: the case for quadruple-H therapy." Anaesthesia and intensive care 42.1 (2014): 78.
A 35-year-old female with no known previous medical history presents to the emergency department with a decreased conscious level.
The following results are obtained
Venous Biochemistry |
|||||
Parameter |
Patient Value |
Normal Adult Range |
|||
Sodium |
144 mmol/L |
135 – 145 |
|||
Potassium |
4.0 mmol/L |
3.5 – 4.5 |
|||
Chloride |
100 mmol/L |
95 |
– |
105 |
|
Bicarbonate |
14 mmol/L* |
22 |
– |
26 |
|
Glucose |
1.1mmol/L* |
3.5 – 6.1 |
|||
Urea |
2.7 mmol/L |
2.9 – 8.2 |
|||
Creatinine |
120 μmol/L |
70 |
– |
120 |
|
Albumin |
46 g/L |
35 |
– |
55 |
|
Total bilirubin |
90 μmol/L* |
< 20 |
|||
Alkaline phosphatase (ALP) |
131 U/L* |
36 |
– |
92 |
|
Aspartate aminotransferase (AST) |
2450 U/L* |
< 40 |
|||
Gamma glutamyl transferase (GGT) |
50 U/L* |
< 30 |
|||
Alanine aminotransferase (ALT) |
2750 U/L* |
< 35 |
|||
Coagulation Tests |
|||||
Parameter |
Patient Value |
Normal Adult Range |
|||
PT |
45 sec* |
12 |
– |
16 |
|
APTT |
46 sec* |
25.0 |
– 37.0 |
||
Fibrinogen |
0.2 g/ L* |
2.20 |
– 4.30 |
a) Give one diagnosis which will explain the clinical and laboratory findings. (10% marks)
b) List six possible aetiologies. (20% marks)
a)
b)
Well, you can't call it "acute liver failure" or "hyper-acute" or subacute" because these have specific time intervals as definitions. All you can really say is that there is liver failure. The college gives malingant infiltration as a differential, which gives one the impression that they would be willing to consider the more insidious causes of liver damage.
Possible categories of the causes:
Vascular:
Infectious:
Neoplastic:
Drug-induced:
|
Idiopathic: idiosyncratic drug reactions:
Congenital:
Autoimmune:
Traumatic:
Endocrine/metabolic:
|
A 42-year-old male is admitted to ICU following a cadaveric orthotopic liver transplant for end-stage liver disease secondary to alcohol-induced cirrhosis.
a) List the important management principles for the first 24 hours specific to this patient. (70% marks)
b) Despite weaning sedation he remains unresponsive 12 hours after ICU admission. What are the possible causes? (30% marks)
a)
Haemodynamic stabilization – optimize cardiac output and tissue perfusion and avoid fluid overload as ventricular function may be impaired. Close haemodynamic monitoring. Vaso-active agents as indicated.
Correction of anaemia and coagulopathy – maintain haemocrit 0.25 – 0.3 to keep blood viscosity low. INR 2, APTT 50 secs, Fibrinogen above 0.5 g/L and Platelets above 30 x 109/L.
Fluid and electrolyte management – appropriate negative fluid balance day 1 decreases risk of pulmonary complications. Fluid overload may aggravate graft congestion and oedema caused by ischaemic-reperfusion. Electrolyte imbalances are common and need to be corrected.
Correction of metabolic abnormalities – hypoglycaemia is an ominous sign of compromised liver recovery, hyperglycaemia also may occur, acid-base abnormalities also occur
Early weaning from mechanical ventilation – associated with better outcome but not feasible in patients with respiratory failure, haemodynamic instability, pulmonary oedema, primary graft dysfunction, encephalopathy etc. Unsuccessful early extubation may result in impaired oxygen delivery to transplanted liver
Monitoring of graft function LFTs, lactate, BSL, coagulation, hepatic artery doppler
Early detection of surgical complications - bleeding
Immunosuppressants
Infection prophylaxis
Housekeeping including analgesia (PCA) and appropriate nutrition plan
Other – ICP monitoring if decompensated CLD pre-op
b)
Delayed metabolism of sedative / anaesthetic drugs
Metabolic derangements – hypoglycaemia, hyponatraemia, hyperosmolar syndrome Hepatic encephalopathy
Hypoxic-ischaemic cerebral injury
Seizures
Intracerebral haemorrhage
This question is essentially identical to Question 11 from the second paper of 2012. The answer has been reproduced below, in the face of goof SEO but in the interest of siplified revision.
There is also plenty of literature regarding the critical care management of liver transplant recipients.
Let us approach this answer systematically.
The following management steps may be followed in the first 24 hour period:
The second part of this question is far less interesting, and refers mainly to the candidate's ability to generate differentials for a decreased level of consciousness.
Using a usual template, one can arrive at a series of differential easily:
The best, most comprehensive source is this article:
Feltracco, Paolo, et al. "Intensive care management of liver transplanted patients." World journal of hepatology 3.3 (2011): 61.
The rest are also helpful.
Stieber, Andrei C., R. D. Gordon, and J. R. Galloway. "Orthotopic liver transplantation." Hepatology. A text book of liver disease. WB Saunders, Philadelphia (1996): 1759-1780.
Mazariegos, George V., Ernesto P. Molmenti, and David J. Kramer. "Early complications after orthotopic liver transplantation." Surgical Clinics of North America 79.1 (1999): 109-129.
Razonable, Raymund R., et al. "Critical care issues in patients after liver transplantation." Liver Transplantation 17.5 (2011): 511.
Mandell, M. Susan, et al. "Reduced use of intensive care after liver transplantation: influence of early extubation." Liver Transplantation 8.8 (2002): 676-681.
A 70-year-old male presents to the ED with a 2-week history of increasing dyspnoea, cough with altered sputum and fever. Past history includes chronic obstructive airways disease (COPD), lung cancer seven years ago treated with chemotherapy and radiation therapy with no sign of recurrence since.
Examination findings included RR 30 breaths/min, BP 110/70mmHg, HR 145 bpm, Temp 37.4ºC, anxious and distress but tired and peripherally cold and cyanosed.
CXR shows findings consistent with COPD and right lower lobe infiltrate.
The following arterial blood gas is taken one hour after receiving 2 litres of fluid resuscitation, antibiotics and bi-level non-invasive ventilation (NIV), at FiO2 = 1.0.
(This blood gas is discussed in Question 23.1 )
Parameter
Patient Value
Normal Adult Range
FiO2
1.0
pH
7.16*
7.35
– 7.45
PCO2
33 mmHg* (4.3 kPa)*
35
–
45 (4.6 – 6.0)
PO2
272 mmHg (38.5 kPa)
Bicarbonate
11 mmol/L*
22
–
30
Base Excess
-17 mmol/L*
-3 – +3
Sodium
138 mmol/L
135 – 145
Potassium
4.3 mmol/L
3.5 – 5.0
Chloride
121 mmol/L*
95
–
110
Glucose
13.1 mmol/L*
3.5 – 7.8
Lactate
6.4 mmol/L*
0.6 – 2.4
Haemoglobin
131 g/L*
135 – 175
Creatinine
150 micromol/L*
70
–
120
Six hours later the patient remains on NIV, is conscious, reports feeling slightly better, feet remain cyanosed, BP 105/72 mmHg, HR 108 bpm, RR 30 breaths/min, urine output 10 – 20 mL/hr and the following biochemistry profile is obtained:
Parameter |
Patient Value |
Normal Adult Range |
||||
Sodium |
139 mmol/L |
135 – 145 |
||||
Potassium |
5.5 mmol/L* |
3.5 – 5.2 |
||||
Chloride |
110 mmol/L |
95 |
– 110 |
|||
Bicarbonate |
12 mmol/L* |
22 |
– 32 |
|||
Urea |
20.0 mmol/L* |
2.7 – 7.8 |
||||
Creatinine |
220 μmol/L* |
70 |
– 120 |
|||
Estimated glomerular filtration rate (eGFR) |
25 mL/min/1.73 m2* |
> 90 |
||||
Anion gap |
22 mmol/L* |
8 – 18 |
||||
Total protein |
57 g/L* |
60 |
– 80 |
|||
Albumin |
27 g/L* |
35 |
– 50 |
|||
Total bilirubin |
24.9 μmol/L |
< 25 |
||||
Alkaline phosphatase (ALP) |
81 IU/L |
30 |
– 110 |
|||
Alanine transaminase (ALT) |
6138 IU/L* |
< 65 |
||||
Aspartate transaminase (AST) |
10122 IU/L* |
< 50 |
||||
g-Glutamyl transferase (GGT) |
88 IU/L |
< 90 |
||||
C-reactive protein (CRP) |
22.5 mg/L* |
< 8 |
b) Give your interpretation of these findings. Include likely aetiologies. (40% marks)
Increasing anion gap due to worsening renal impairment and possibly increasing lactate.
LFTs deranged with predominant finding of transaminitis. (This is likely to be associated with an increase in lactate).
Aetiologies
The following biochemical abnormalities are present in the second set of results:
The raised transaminases are described as a "transaminitis" by the college, a term which has been in use since 1977, in spite of some authors referring to it as "made up and improper" (Stellpflug, 2011). The previous set of results from Question 23.1 demonstrated a raised lactate and severe acidaemia wioth a normal glucose and normal haemoglobin. From the list of causes of elevated transaminases, the following are relevant diferentials, and parenthesised comments as to why they are relevant:
And then those things which this is not:
Stellpflug, Samuel J. "Transaminitis: The Lab Test That Has Inflammation…." Journal of Medical Toxicology 7.3 (2011): 252-253.
Johnston, David E. "Special considerations in interpreting liver function tests." American family physician 59 (1999): 2223-2232.
Limdi, J. K., and G. M. Hyde. "Evaluation of abnormal liver function tests." Postgraduate medical journal 79.932 (2003): 307-312.
Hoekstra, Lisette T., et al. "Physiological and biochemical basis of clinical liver function tests: a review." Annals of surgery 257.1 (2013): 27-36.
Outline the risk factors and your management strategies for constipation in the critically ill.
Risk factors
Management Strategies
Non-pharmacological Interventions:
Pharmacological Interventions:
Risk factors for constipation in the critically ill:
Modifiable ICU-related risk factors
Non-modifiable disease risk factors
Management strategies for constipation in the critically ill
Tier one therapies and investigations:
Tier two therapies and investigations: Assuming bowel obstruction and megacolon are ruled out
Tier three therapies and investigations: assuming all of the above are ineffective, or mpossible to implement
Mostafa, S. M., et al. "Constipation and its implications in the critically ill patient†." British journal of anaesthesia 91.6 (2003): 815-819.
Nassar, Antonio Paulo, Fernanda Maria Queiroz da Silva, and Roberto de Cleva. "Constipation in intensive care unit: incidence and risk factors." Journal of critical care 24.4 (2009): 630-e9.
Blaser, Annika Reintam, et al. "Gastrointestinal function in intensive care patients: terminology, definitions and management. Recommendations of the ESICM Working Group on Abdominal Problems." Intensive care medicine 38.3 (2012): 384-394.
Artinyan, Avo, et al. "Prolonged postoperative ileus—definition, risk factors, and predictors after surgery." World journal of surgery 32.7 (2008): 1495-1500.
Nadrowski, L. "Paralytic ileus: recent advances in pathophysiology and treatment." Current surgery 40.4 (1982): 260-273.
Smonig, Roland, et al. "Constipation is independently associated with delirium in critically ill ventilated patients." Intensive Care Medicine 42.1 (2016): 126-127.
Patanwala, Asad E., et al. "Pharmacologic management of constipation in the critically ill patient." Pharmacotherapy: The Journal of Human Pharmacology and Drug Therapy 26.7 (2006): 896-902.
Gacouin, Arnaud, et al. "Constipation in long-term ventilated patients: associated factors and impact on intensive care unit outcomes." Critical care medicine 38.10 (2010): 1933-1938.
Azevedo, Rodrigo Palácio de, and Flávia Ribeiro Machado. "Constipation in critically ill patients: much more than we imagine." Revista Brasileira de terapia intensiva 25.2 (2013): 73-74.
de Souza Guerra, Tatiana Lopes, Simone Sotero, and Norma Guimarães Marshall Mendonça. "Incidência de constipação intestinal em uma unidade de terapia intensiva." Rev Bras Ter Intensiva 25.2 (2013): 87-92.
A 45-year-old male with a background of chronic liver disease is admitted to the Emergency Depattment (ED) with massive haematemesis secondary to gastric varices. He is managed with endoscopy and sclerotherapy.
List four other causes for massive haematemesis. (10% marks)
List the clinical indicators for risk of re-bleeding from the gastric varices. (20% marks)
List the pharmacological agents that may reduce the risk of a re-bleed. (20% marks)
Following initial stabilisation and control of bleeding, he deteriorates with a variceal re-bleed.
List the options for controlling the re-bleed AND, where appropriate, the relative advantages and disadvantages of these. (50% marks)
a) Causes
• Gastric or duodenal ulcer with bleeding visible vessel
• Dieulafoy's lesion (large exposed arteriole within gastric wall)
• Tear at gastro-oesophageal junction (Mallory Weiss)
• Aorto-duodenal fistula
• Eroding cancer into vessel (short gastric artery, splenic artery)
b) Rebleed likely if:
• Advanced age
• Unable to band all varices
• Gastric > oesophageal varices
• Severe coagulopathy due to liver disease or massive transfusion
• Severity of portal hypertension or liver disease
• Size of varices – larger higher risk
• Presence of red signs (localised reddish spots on the mucosal surface of the varix)
c) Drugs to reduce risk of re-bleed
• Octreotide/somatostatin
• Vasopressin / terlipressin +/- venodilator
• Tranexamic acid
• Oral Sucralfate (local anti-fibrinolytic effect)
• PPI infusion if concomitant ulcer bleeding
• Beta blockers e.g. propranolol if haemodynamics permit
• Short-term prophylactic antibiotics
d) Options for re-bleeding:
• Measure and fix coagulation, ongoing resuscitation
• Repeat endoscopy
o Can be done in ICU although may be more appropriate in the operating theatre
o Requires airway protection
o Allows endoscopic variceal obturation or endoscopic variceal ligation
• TIPS to reduce portal pressure; risks of encephalopathy
o Strategy of choice with initial treatment failure
o May be contra-indicated in high MELD score
o Complications of shunting blood away from liver and increased hepatic encephalopathy
• Balloon tamponade (Sengstaken, Minnesota)
o Only useful in varices in the oesophagus or GO junction; not useful for gastric
o Requires airway protection
o Mucosal injury and necrosis
• Surgery
o Ligation and resection of gastric vessels
o Oesophageal venous ligation
▪ Requires luminal incision; high risk of breakdown in context of liver disease
▪ May not be available depending on local resources
• Balloon-occluded retrograde transverse obliteration (BRTO)
o New technique and still undergoing evaluation
o Increases portal hepatic blood flow and may be alternative for patients who may not tolerate TIPS
o Obliterates spontaneous porto-systemic shunts and may aggravate portal hypertension
• Activated factor 7
o Questionable efficacy
o Highly pro-coagulant
o May have a role in buying time to allow retrieval to a more specialised centre
a) Other causes of haematemesis could include a whole range of differentials. Owing to the author's shameful obsession with structured classifications, this range can be divided into "blood is coming from the gut" and "blood is not coming from the gut but somehow has ended up in the gut". This was generated with the use of Oh's Manual, but contains conditions which are not listed in the canonic Chapter 42 (pp. 487, "Acute gastrointestinal bleeding" by Joseph JY Sung).
Bleeding of gastrointestinal origin
Bleeding of non-gastrointestinal origin
b) The college have listed causes of rebleeding which seem fairly logical, eg. "unable band everything" and "still coagulopathic". To this list, one might also add gastroenterological lazyness (delayed endoscopy increases re-bleeding risk according to Chen et al, 2012). From Augustine et al (2010), there are several features found to be strongly associated with "five-day failure", a composite endpoint consisting of re-bleeding and five-day mortality. All of these have been combined into this list:
c) Pharmacological agents which can decrease the risk of rebleeding:
d) Options for controlling a re-bleed: whenever the college ask for something with a list of advantages and disadvantages, it is usually better to put it in a table. Thus:
Strategy | Advantages | Disadvantages |
Medical resuscitation |
|
|
Repeat endoscopy |
|
|
Balloon tamponade |
|
|
TIPS |
|
|
Surgical control |
|
|
Balloon-occluded retrograde transverse obliteration (BRTO) |
|
|
Factor VIIa is also mentioned by the college in their answer, though they themselves moderate their enthusiasm by pointing out that it has "questionable efficacy". If one reads the papers (eg. Bosch et al, 2008) this would certainly seem correct (there was no effect on any primary endpoints). The mention of this option in the college answer is itself questionable, as it is promoted as an option for controlling a re-bleed even when trial results "do not support the routine use of rFVIIa in this setting". In protest, I did not add it to my table.
Oh's Intensive Care manual: Chapter 42 (pp. 487) Acute gastrointestinal bleeding by Joseph JY Sung
arcia-Tsao, Guadalupe, and Jaime Bosch. "Management of varices and variceal hemorrhage in cirrhosis." New England Journal of Medicine 362.9 (2010): 823-832.
García-Pagán, Juan Carlos, et al. "Early use of TIPS in patients with cirrhosis and variceal bleeding." New England Journal of Medicine 362.25 (2010): 2370-2379.
Vlavianos, P., et al. "Balloon tamponade in variceal bleeding: use and misuse."BMJ: British Medical Journal 298.6681 (1989): 1158.
Reverter, Enric, and Juan Carlos García‐Pagán. "Management of an acute variceal bleeding episode." Clinical Liver Disease 1.5 (2012): 151-154.
Ioannou, G. N., J. Doust, and D. C. Rockey. "Terlipressin in acute oesophageal variceal haemorrhage." Alimentary pharmacology & therapeutics 17.1 (2003): 53-64.
Corley, Douglas A., et al. "Octreotide for acute esophageal variceal bleeding: a meta-analysis." Gastroenterology 120.4 (2001): 946-954.
Reiberger, Thomas, et al. "Carvedilol for primary prophylaxis of variceal bleeding in cirrhotic patients with haemodynamic non-response to propranolol." Gut62.11 (2013): 1634-1641.
Hou, Ming‐Chih, et al. "Antibiotic prophylaxis after endoscopic therapy prevents rebleeding in acute variceal hemorrhage: a randomized trial." Hepatology 39.3 (2004): 746-753.
Augustin, Salvador, Antonio González, and Joan Genescà. "Acute esophageal variceal bleeding: Current strategies and new perspectives." World J Hepatol 2.7 (2010): 261-274.
Chen, Ping-Hsien, et al. "Delayed endoscopy increases re-bleeding and mortality in patients with hematemesis and active esophageal variceal bleeding: a cohort study." Journal of hepatology 57.6 (2012): 1207-1213.
Kleber, Gerhard, et al. "Prediction of variceal hemorrhage in cirrhosis: a prospective follow-up study." Gastroenterology 100.5 (1991): 1332-1337.
A 51 -year-old female presents with a decreased conscious state, Glasgow Coma Scale (GCS) 12, confusion and myoclonus. She is on treatment for a seizure disorder. Her CT brain scan shows no acute intracranial abnormality.
Her investigations are as follows:
Parameter |
Patient Value |
Adult Normal Range |
|
Sodium |
1 38 mmol/L |
135-145 |
|
Potassium |
4.1 mmollL |
3.5 - 5.2 |
|
Bicarbonate |
18 mmol/l_• |
22 - 32 |
|
Urea |
14.2 mmoVL• |
3.0 - 8.0 |
|
Creatinine |
210 mol/l_• |
45 - 90 |
|
Bilirubin |
54 mol/L* |
< 20 |
|
Alanine transferase |
2710 |
< 35 |
|
Aspartate transferase |
1365 |
< 35 |
|
Alkaline phosphatase |
103 Ull- |
30- 110 |
|
Glutam transferase |
67 U/L* |
< 40 |
|
Albumin |
37 |
35-50 |
|
Protein |
61 IL |
60 - 80 |
|
Ammonia |
156 |
< 50 |
|
List possible causes of the hyper-ammonaemia in this patient. (40% marks)
The patient; she is probably on valproate for her seizure disorder. Valproate overdose is a logical explanation for a decreased level of consciousness, myoclonus and high ammonia.
Causes of hyperammonaemia more broadly could be any of the following:
Increased substrate for ammoniagenesis
Bypass of normal metabolism
|
Acquired urea cycle defects
Congenital urea cycle defects
Excess of exogenous ammonia
Reabsorption of excreted ammonia
|
Conway, Edward Joseph, and Robert Cooke. "Blood ammonia." Biochemical Journal 33.4 (1939): 457.
Shambaugh, G. E. "Urea biosynthesis I. The urea cycle and relationships to the citric acid cycle." The American journal of clinical nutrition 30.12 (1977): 2083-2087.
McDermott Jr, William V., Raymond D. Adams, and Athol G. Riddell. "Ammonia metabolism in man." Annals of surgery 140.4 (1954): 539.
Vince, Angela, et al. "Ammonia production by intestinal bacteria." Gut 14.3 (1973): 171-177.
Vince, Angela J., and Sigrid M. Burridge. "Ammonia production by intestinal bacteria: the effects of lactose, lactulose and glucose." Journal of medical microbiology 13.2 (1980): 177-191.
Dohrenwend, Paul, and Richard D. Shih. "Glycine Induced Hyperammonemia After Bladder Rupture During Transurethral Resection of a Bladder Tumor." Journal of Medical Cases 4.4 (2013): 250-253.
Felipo, Vicente, and Roger F. Butterworth. "Neurobiology of ammonia." Progress in neurobiology 67.4 (2002): 259-279.
Hashim, Ibrahim A., and Jennifer A. Cuthbert. "Elevated ammonia concentrations: Potential for pre-analytical and analytical contributing factors." Clinical biochemistry 47.16 (2014): 233-236.
Clay, Alison S., and Bryan E. Hainline. "Hyperammonemia in the ICU." CHEST Journal 132.4 (2007): 1368-1378.
Weng, Te-I., Frank Fuh-Yuan Shih, and Wen-Jone Chen. "Unusual causes of hyperammonemia in the ED." The American journal of emergency medicine 22.2 (2004): 105-107.
Hawkes, N. D., et al. "Non-hepatic hyperammonaemia: an important, potentially reversible cause of encephalopathy." Postgraduate medical journal 77.913 (2001): 717-722.
Outline the pathophysiology, diagnosis and treatment of mesenteric ischaemia.
Mesenteric ischaemia occurs when blood flow is inadequate to meet the metabolic demands of the small bowel or colon.
Pathophysiology
Diagnosis
Treatment
Additional Examiners’ Comments:
The template above is only a guide to the expected answer.Important points sought by the Examiners were: the different categories of mesenteric ischaemia, comments about importance of history, examination and suspicion; it was essential to mention surgery as a diagnostic tool.
The template below is also only a guide.
Pathophysiology
History
Examination
Biochemistry
Imaging
Specific management
Supportive management
Acosta, Stefan, and Martin Björck. "Modern treatment of acute mesenteric ischaemia." British Journal of Surgery 101.1 (2014).
Schofield, Nick, et al. "Acute mesenteric ischaemia." Journal of the Intensive Care Society 15.3 (2014): 226-230.
Clair, Daniel G., and Jocelyn M. Beach. "Mesenteric ischemia." New England Journal of Medicine 374.10 (2016): 959-968. (pdf)
Pecoraro, Felice, et al. "Chronic mesenteric ischemia: critical review and guidelines for management." Annals of vascular surgery 27.1 (2013): 113-122.
You have been asked to review a 53-year-old female with known alcoholic liver disease. She has had a progressive fall in her conscious level over the last 24 hours and the medical team are concerned she is developing hepatic encephalopathy (HE).
a) List four alternative diagnoses to HE that you would consider in this circumstance. (10% marks)
b) List six clinical signs that would be suggestive of HE. (30% marks)
c) Discuss the specific management of severe HE in this setting. (60% marks)
a.
• Drug or alcohol effects
• Seizure disorder
• Traumatic injury
• Septic encephalopathy
• Hypoglycaemia and other electrolyte disorders
• Uraemia
b.
• Asterixis
• Hypertonia
• Hyperreflexia
• Clonus
• Hippus
• Bradykinesia
• Nystagmus
c.
c.
• Basic principles of management include:
Excluding other causes of altered mentation
HE is a diagnosis of exclusion (part [a] of question 10). Serum ammonia levels should not be used as a diagnostic tool or to monitor response to treatment.
Identifying and treating precipitating cause
• Increased nitrogen load (GI bleed, excess protein intake, infection)
• Decreased toxin clearance (hypovolaemia, renal failure, constipation, port-systemic shunt, medication non-compliance, acute on chronic liver failure)
• Altered neurotransmission (sedating medications, alcohol, hypoxia, hypoglycaemia)
Reducing nitrogen load in the gut/Ammonia formation
First Line-
• Non-absorbable disaccharidases- Lactulose is metabolized by bacteria in the colon to acetic and lactic acid, which reduces colonic pH, decreases survival of urease producing bacteria in the gut, and facilitates conversion of ammonia (NH3) to ammonium (NH4+), which is less readily absorbed by the gut. The cathartic effect of these agents also increases faecal nitrogen waste.
• Antibiotics-Rifaximin is a minimally absorbed oral antibiotic with broad spectrum activity against gram-positive and gram-negative aerobic and anaerobic bacteria. Oral neomycin and metronidazole have been used to treat hepatic encephalopathy in the past but due to concerns of toxicity and side effects, rifaximin is now the preferred antibiotic.
Second line- (all not required to score full marks)
• Probiotics- As gut bacteria play a central role in producing ammonia it has been theorized that altering gut flora using probiotics may be beneficial in HE.
• Polyethylene Glycol- Commonly used, safe and highly effective laxative that has recently been proposed as a possible agent for HE
• Flumazenil- benzodiazepine antagonist at GABA receptors. Can result in clinical improvement but no mortality benefit
• Ammonia scavengers- increase ammonia clearance and reduce systemic concentrations by providing an alternative pathway for renal ammonia clearance.
• L-ornithine l-aspartate (LOLA)- increases glutamine synthase and urea excretion. Similar clinical improvement when compared to lactulose.
• Zinc- Low zinc is associated with impairment of urea cycle enzymes leading to elevated ammonia levels.
• Porta-systemic shunts- Medically refractory HE should raise suspicion of a spontaneous splenorenal shunt and patients who have undergone TIPS should be considered for shunt reversal if severe HE persists
Supportive care
Management of cerebral oedema-Lactulose or rifaximin can be beneficial for the treatment of gradual-onset encephalopathy in patients with prior cirrhosis, but additional, aggressive treatment of brain edema with osmotic diuretics is required in new, fulminant forms to prevent secondary, permanent brain-stem damage and to sustain patients through liver transplantation.
Nutrition - plays a key role in managing HE and preventing recurrence. Optimal daily energy intake should be 35 to 40 kcal/kg ideal body weight with daily protein intake of 1.2 to 1.5 g/kg ideal body weight.Multivitamin should be considered with the addition of specific treatments for clinically apparent vitamin deficiencies.
Assessing the need for long term therapy and liver transplant evaluation
Liver transplant evaluation should be considered in appropriate candidates once a diagnosis of overt hepatic encephalopathy is made
Possible causes of unconsciousness in the middle-aged alcoholic could be anything, considering also that we are not given any better description of it (i.e. it could represent merely confusion rather than coma).
The top four would have to be:
Other possibilities include:
Clinical signs which might suggest HE can be extracted from this ancient grading system which was developed by Conn and Lieberthal (1979)
Stage I |
|
Stage II |
|
Stage III |
|
Stage IV |
|
Specific steps in the management of hepatic encephalopathy should consist of specific therapies targeted at reducing the hepatic toxin load and management of the precipitating cause. The excellent college answer offers some detailed discussion, which is difficult to improve upon.
J.S. Bajaj (2010) offers a detailed exploration of the modern management of hepatic encephalopathy. Another excellent review is from Riggio et al (2015). From these, the following approach has been concocted, to easily recall and regurgitate in an exam setting:
Specific management of hepatic encephalopathy
|
Management of the precipitating cause
|
Supportive management of the encephalopathic patient
|
Chapter 49 (pp. 549) Disorders of consciousness by Balasubramanian Venkatesh
Chapter 44 (pp. 501) Liver failure by Christopher Willars and Julia Wendon
Blei, Andres T., and Juan Córdoba. "Hepatic encephalopathy." The American journal of gastroenterology 96.7 (2001): 1968-1976.
Ferenci, Peter, et al. "Hepatic encephalopathy—definition, nomenclature, diagnosis, and quantification: final report of the working party at the 11th World Congresses of Gastroenterology, Vienna, 1998." Hepatology 35.3 (2002): 716-721.
Prakash, Ravi, and Kevin D. Mullen. "Mechanisms, diagnosis and management of hepatic encephalopathy." Nature Reviews Gastroenterology and Hepatology7.9 (2010): 515-525.
Nabi, Eiman, and Jasmohan S. Bajaj. "Useful Tests for Hepatic Encephalopathy in Clinical Practice." Current gastroenterology reports 16.1 (2014): 1-8.
Guillén, Juan C. Quero, and Juan M. Herrerías Gutiérrez. "Diagnostic methods in hepatic encephalopathy." Clinica chimica acta 365.1 (2006): 1-8.
Shawcross, Debbie L., and Julia A. Wendon. "The neurological manifestations of acute liver failure." Neurochemistry international 60.7 (2012): 662-671.
Conn, Harold O., and Milton M. Lieberthal. The hepatic coma syndromes and lactulose. Williams & Wilkins, 1979. - this is a book, and not available for free online.
With respect to the management of patients presenting with acute pancreatitis, briefly discuss the following issues:
a) The optimal timing and method of delivery of nutrition.(40% marks)
b) The role of antimicrobials.(40% marks)
c) The role of endoscopic retrograde cholangio-pancreatography (ERCP).(20% marks)
(a) Method of delivery of nutrition
(b) Use of antimicrobials
(c) Role of ERCP
Nutrition in acute pancreatitis:
Antibiotics in pancreatitis:
The role of ERCP in pancreatitis:
"May be cause", the college say economically, as if to type a "the" into their answer would incur an intolerable time cost.
Specifically, section L of the 2016 statement
ESPEN guidelines :
specifically,
MACFIE, J., and ESPEN CONSENSUS GROUP. "ESPEN guidelines on nutrition in acute pancreatitis." Clinical Nutrition 21.2 (2002): 173-183.
Eatock FC, Chong P, Menezes N, Murray L, McKay CJ, Carter CR, Imrie CW. A randomized study of early nasogastric versus nasojejunal feeding in severe acute pancreatitis. Am J Gastroenterol. 2005 Feb;100(2):432-9.
Windsor AC, Kanwar S, Li AG, et al. Compared with parenteral nutrition, enteral feeding attenuates the acute phase response and improves disease severity in acute pancreatitis. Gut. 1998;42: 431-435.
Ragins H, Levenson SM, Signer R, Stamford W, Seifter E Intrajejunal administration of an elemental diet at neutral pH avoids pancreatic stimulation. Studies in dog and man. .Am J Surg. 1973 Nov;126(5):606-14.
B. W. M. Spanier,1, M. J. Bruno, E. M. H. Mathus-Vliegen Enteral Nutrition and Acute Pancreatitis: A Review Gastroenterol Res Pract. 2011; 2011: 857949. Published online 2010 August 3.
Casaer, Michael P., et al. "Early versus late parenteral nutrition in critically ill adults." N Engl J Med 365.6 (2011): 506-517.
Abou-Assi, Souheil, Kimberly Craig, and Stephen JD O’Keefe. "Hypocaloric jejunal feeding is better than total parenteral nutrition in acute pancreatitis: results of a randomized comparative study." The American journal of gastroenterology 97.9 (2002): 2255-2262.
Petrillo-Albarano, Toni, et al. "Use of a feeding protocol to improve nutritional support through early, aggressive, enteral nutrition in the pediatric intensive care unit*." Pediatric Critical Care Medicine 7.4 (2006): 340-344.
Mirtallo, Jay M., et al. "International consensus guidelines for nutrition therapy in pancreatitis." Journal of Parenteral and Enteral Nutrition (2012): 0148607112440823.
Al-Omran, Mohammed, Ala Groof, and Derek Wilke. "Enteral versus parenteral nutrition for acute pancreatitis." Cochrane Database Syst Rev 1.1 (2003).
Ho, Kwok M., Geoffrey J. Dobb, and Steven AR Webb. "A comparison of early gastric and post-pyloric feeding in critically ill patients: a meta-analysis." Intensive care medicine32.5 (2006): 639-649.
Chang, Yu-sui, et al. "Nasogastric or nasojejunal feeding in predicted severe acute pancreatitis: a meta-analysis." Critical Care 17.3 (2013): R118.
Petrov, Maxim S., M. Isabel TD Correia, and John A. Windsor. "Nasogastric tube feeding in predicted severe acute pancreatitis. A systematic review of the literature to determine safety and tolerance." JOP. Journal of the Pancreas (2008).
Vaughn, Valerie M., et al. "Early versus delayed feeding in patients with acute pancreatitis: a systematic review." Annals of Internal Medicine 166.12 (2017): 883-892.
Oh's Intensive Care manual: Chapter 43 (pp. 495) Severe acute pancreatitis by Duncan LA Wyncoll
Heinrich, Stefan, et al. "Evidence-based treatment of acute pancreatitis: a look at established paradigms." Annals of surgery 243.2 (2006): 154-168.
Pederzoli, Paolo, et al. "A randomized multicenter clinical trial of antibiotic prophylaxis of septic complications in acute necrotizing pancreatitis with imipenem." Surgery, gynecology & obstetrics 176.5 (1993): 480-483.
Wilmer, Alexander. "ICU management of severe acute pancreatitis." European journal of internal medicine 15.5 (2004): 274-280.
Villatoro, Eduardo, Mubashir Mulla, and Mike Larvin. "Antibiotic therapy for prophylaxis against infection of pancreatic necrosis in acute pancreatitis."Cochrane Database Syst Rev 5.5 (2010).
Mirtallo, Jay M., et al. "International consensus guidelines for nutrition therapy in pancreatitis." Journal of Parenteral and Enteral Nutrition 36.3 (2012): 284-291.
Baltatzis, Minas, et al. "Antibiotic use in acute pancreatitis: Global overview of compliance with international guidelines." Pancreatology (2016).
Tenner, Scott, et al. "American College of Gastroenterology guideline: management of acute pancreatitis." The American journal of gastroenterology 108.9 (2013): 1400-1415.
Gabbrielli, Armando, et al. "ERCP in acute pancreatitis: What takes place in routine clinical practice?." World journal of gastrointestinal endoscopy 2.9 (2010): 308.
Wu, Bechien U., and Peter A. Banks. "Clinical management of patients with acute pancreatitis." Gastroenterology 144.6 (2013): 1272-1281.
Cherian, Jijo V., et al. "ERCP in acute pancreatitis." Hepatobiliary Pancreat Dis Int 6.3 (2007): 233-240.
Zhu, Youfeng, et al. "Nasogastric nutrition versus nasojejunal nutrition in patients with severe acute pancreatitis: a meta-analysis of randomized controlled trials." Gastroenterology research and practice 2016 (2016).
a) What are the radiological features of colonic pseudo-obstruction / Ogilvie's syndrome? (20% marks)
b) List six conditions which are associated with colonic pseudo obstruction (20% marks)
c) Briefly outline your approach to management. (60% marks)
a)
Plain films: - Identical to mechanical obstruction: dilated bowel loops: may have fluid levels CT demonstrates dilated large bowel without a clear transition point or obstructing lesion.
b)
Trauma, especially fractures
Recent surgery, especially involving spinal anaesthesia
Burns
Diabetes Mellitus
Uraemia
Severe medical illness, such as pneumonia, myocardial infarction, or heart failure
Neurologic conditions
Chemotherapy (e.g., all-trans retinoic acid, methotrexate, vincristine)
Retroperitoneal pathology, such as malignancy or haemorrhage
Electrolyte disturbance
Medication (e.g., narcotics, phenothiazine’s, calcium channel blockers, alpha-2-adrenergic agonists, epidural analgesics)
c)
Initial management of acute colonic pseudo-obstruction consists of conservative therapy in patients without significant abdominal pain or signs of peritonitis and those who have one or more potential factors that are reversible.
Treat underlying disease, stop aggravating drugs, avoid laxatives, and keep NPO. NG tube – encourage mobility. Consider opiate reversal agents e.g. GI naloxone or SC Naltrexone
If fail or progress consider neostigmine: In patients with caecal diameter >12 cm (varies) or failure of 24 to 48 hours of conservative therapy. Up to 2 mg slow IV and repeat if needed. Lower doses may also be effective. Studies have shown high response rate with low rate of recurrence. Side effects include abdominal pain, hypersalivation, vomiting and bradycardia. Perforation may occur if there is unrecognised mechanical obstruction.
Colonoscopy decompression: Those patients who fail or who have contraindications to neostigmine. Technically difficult and perforation is a risk. No randomised trials.
Surgery: In the absence of a colonic perforation, cecostomy tube or a segmental or subtotal resection with primary anastomosis can be performed. In the patients with a colonic perforation, a total colectomy, ileostomy, and Hartmann procedure are performed to retain the option of future ileorectal anastomosis
Examiners Comments:
Management plan poorly structured in many cases. Overall reasonably well answered.
a)
Radiological features of colonic pseudo-obstruction:
b)
The list of risk factors for colonic-pseudo-obstruction specifically is derived from Saunders et al (2005) and Wells et al (2017):
For ileus and constipation in general, you would expect the following risk factors:
Modifiable ICU-related risk factors
Non-modifiable disease risk factors
c)
Most of this comes from Saunders et al (2005). Options for management of colonic pseudo-obstruction can be divided into categories:
Supportive
Pharmacological (pro-motility) management
Interventional (decompressive) management
Ogilvie, Heneage. "Large-intestine colic due to sympathetic deprivation." British Medical Journal 2.4579 (1948): 671.
Choi, Ji Soo, et al. "Colonic pseudoobstruction: CT findings." American Journal of Roentgenology 190.6 (2008): 1521-1526.
Saunders, M. D., and M. B. Kimmey. "Systematic review: acute colonic pseudo‐obstruction." Alimentary pharmacology & therapeutics 22.10 (2005): 917-925.
Wells, Cameron I., Gregory O’Grady, and Ian P. Bissett. "Acute colonic pseudo-obstruction: A systematic review of aetiology and mechanisms." World journal of gastroenterology23.30 (2017): 5634.
With regard to gastric ulceration in the ICU:
a) List five risk factors for developing stress related gastric ulceration in ICU patients.
(20% marks)
b) Discuss briefly strategies for prevention of gastrointestinal bleeding resulting from stress ulcers among ICU patients. Include in your answer the available evidence for these. (80% marks)
a)
Risk factors
b)
Strategies for prevention of GI bleeding resulting from stress ulcers among ICU patients.
H2 blockers:
Inhibits histamine stimulated acid secretion and are better than placebo, antacid or sucralfate as stress ulcer prophylaxis. No evidence they are superior than PPI.
Tolerance, requires dose adjustment in renal failure; rarely causes thrombocytopenia.
Proton pump inhibitors:
Pantoprazole and omeprazole do seem to have some benefit in protecting patients from stress ulceration. In critically ill patients, proton pump inhibitors seem to be more effective than histamine 2 receptor antagonists in preventing clinically important and overt upper gastrointestinal bleeding. No clear evidence that one PPI is better than the other.
Meta-analysis of 13 RCTs; n = 1587 patients (H2 blockers versus PPI) Found less GI bleeding among those who received a PPI (1.3 versus 6.6 percent, odds ratio 0.30, 95% CI 0.17-0.54) no difference in mortality or the incidence of nosocomial pneumonia. (Detail not required)
Side effects include: Interstitial nephritis Clostridium difficile enterocolitis, GI upset and headaches. Long-term use associated with fractures, hypomagnesemia hypocalcemia.
Enteral feeding
Observational studies data suggest that enteral nutrition may be adequate substitute for pharmacologic stress ulcer prophylaxis in ICU patients, however controlled trials are necessary for confirmation. There appears to be no benefit for stress ulcer prophylaxis in patients who are tolerating enteral feeding, and in these patients stress ulcer prophylaxis may not be needed. However, it is still unclear if enteral feeding is alone sufficient in protection of stress ulcers in high risk patients.
Examiners Comments:
Generally, poorly answered. Superficial knowledge of pharmacology and evidence base.
a)
According to Cook et al (1994), independent risk factors for stress ulceration are:
b)
To "include in your answer the available evidence for these", this answer to Question 1 from the first paper of 2003 was updated with contemporary data.
Oh's Intensive Care manual: Chapter 42 (pp. 487) Acute gastrointestinal bleeding by Joseph JY Sung
Cook, Deborah J., et al. "Risk factors for gastrointestinal bleeding in critically ill patients." New England Journal of Medicine 330.6 (1994): 377-381.
Marik, Paul E., et al. "Stress ulcer prophylaxis in the new millennium: a systematic review and meta-analysis." Crit Care Med 38.11 (2010): 2222-2228.
Krag, Mette, et al. "Stress ulcer prophylaxis versus placebo or no prophylaxis in critically ill patients." Intensive care medicine 40.1 (2014): 11-22.
Madsen, Kristian Rørbæk, et al. "Guideline for Stress Ulcer Prophylaxis in the Intensive Care Unit." Danish medical journal 61.3 (2014): 1-4.
Plummer, Mark P., Annika Reintam Blaser, and Adam M. Deane. "Stress ulceration: prevalence, pathology and association with adverse outcomes."Critical Care 18.2 (2014): 213.
Yearsley, K. A., et al. "Proton pump inhibitor therapy is a risk factor for Clostridium difficile‐associated diarrhoea." Alimentary pharmacology & therapeutics 24.4 (2006): 613-619.
Steinberg, Kenneth P. "Stress-related mucosal disease in the critically ill patient: risk factors and strategies to prevent stress-related bleeding in the intensive care unit." Critical care medicine 30.6 (2002): S362-S364.
Buendgens, Lukas, Alexander Koch, and Frank Tacke. "Prevention of stress-related ulcer bleeding at the intensive care unit: Risks and benefits of stress ulcer prophylaxis." World journal of critical care medicine 5.1 (2016): 57.
Gonzalez, Edgar R., and Anthony R. Morkunas. "Prophylaxis of stress ulcers: Antacid titration vs. histamine2-receptor blockade." Drug intelligence & clinical pharmacy 19.11 (1985): 807-811.
Cook, Deborah, et al. "A comparison of sucralfate and ranitidine for the prevention of upper gastrointestinal bleeding in patients requiring mechanical ventilation." New England Journal of Medicine 338.12 (1998): 791-797.
Oh's Intensive Care manual: Chapter 42 (pp. 487) Acute gastrointestinal bleeding by Joseph JY Sung
Cook, Deborah J., et al. "Risk factors for gastrointestinal bleeding in critically ill patients." New England Journal of Medicine 330.6 (1994): 377-381.
Marik, Paul E., et al. "Stress ulcer prophylaxis in the new millennium: a systematic review and meta-analysis." Crit Care Med 38.11 (2010): 2222-2228.
Krag, Mette, et al. "Stress ulcer prophylaxis versus placebo or no prophylaxis in critically ill patients." Intensive care medicine 40.1 (2014): 11-22.
Madsen, Kristian Rørbæk, et al. "Guideline for Stress Ulcer Prophylaxis in the Intensive Care Unit." Danish medical journal 61.3 (2014): 1-4.
Plummer, Mark P., Annika Reintam Blaser, and Adam M. Deane. "Stress ulceration: prevalence, pathology and association with adverse outcomes."Critical Care 18.2 (2014): 213.
Yearsley, K. A., et al. "Proton pump inhibitor therapy is a risk factor for Clostridium difficile‐associated diarrhoea." Alimentary pharmacology & therapeutics 24.4 (2006): 613-619.
Steinberg, Kenneth P. "Stress-related mucosal disease in the critically ill patient: risk factors and strategies to prevent stress-related bleeding in the intensive care unit." Critical care medicine 30.6 (2002): S362-S364.
Buendgens, Lukas, Alexander Koch, and Frank Tacke. "Prevention of stress-related ulcer bleeding at the intensive care unit: Risks and benefits of stress ulcer prophylaxis." World journal of critical care medicine 5.1 (2016): 57.
Gonzalez, Edgar R., and Anthony R. Morkunas. "Prophylaxis of stress ulcers: Antacid titration vs. histamine2-receptor blockade." Drug intelligence & clinical pharmacy 19.11 (1985): 807-811.
Cook, Deborah, et al. "A comparison of sucralfate and ranitidine for the prevention of upper gastrointestinal bleeding in patients requiring mechanical ventilation." New England Journal of Medicine 338.12 (1998): 791-797.
MacLaren, Robert, Catherine L. Jarvis, and Douglas N. Fish. "Use of enteral nutrition for stress ulcer prophylaxis." Annals of Pharmacotherapy 35.12 (2001): 1614-1623.
El-Kersh, Karim, et al. "Enteral nutrition as stress ulcer prophylaxis in critically ill patients: A randomized controlled exploratory study." Journal of critical care 43 (2018): 108-113.
Alhazzani, Waleed, et al. "Withholding pantoprazole for stress ulcer prophylaxis in critically ill patients: a pilot randomized clinical trial and meta-analysis." Critical care medicine 45.7 (2017): 1121-1129.
Barkun, Alan N., et al. "Proton pump inhibitors vs. histamine 2 receptor antagonists for stress-related mucosal bleeding prophylaxis in critically ill patients: a meta-analysis." The American journal of gastroenterology 107.4 (2012): 507.
Outline the specific management issues to address in a patient during the first 24 hours following liver transplantation.
Initial detailed assessment and resuscitation as indicated
Particular care regarding volume status and identification of bleeding and early graft function Adequate analgesia and sedation
Protocolised care; close liaison between ICU and other teams involved e.g. surgeons and transplant physicians
Enteral nutrition
DVT prophylaxis; usually mechanical Early mobilisation
Lines and access management: need to rationalise multiple access when stability achieved and coagulation profile acceptable
Assess suitability for stepdown if no complications
Cardiovascular
Vasodilated state often requiring pressor support for adequate MAP
Careful management of volume status and early recognition of bleeding important; large fluid shifts; drain losses may be large and require ongoing volume administration. Avoid elevated CVP
Graft
Assessment of function via monitoring of coagulation profile, lactate, acid base and transaminases.
(frequent blood tests/QID)
Ultrasound assessment of graft particularly hepatic artery / vein / portal vein patency and flow characteristics
Primary graft nonfunction may be indicated by conventional signs of liver failure i.e. worsening coagulopathy, acidosis, encephalopathy, AKI, hypoglycaemia
Respiratory
Early extubation when stability ensured
Patients with hepatopulmonary syndrome or portopulmonary hypertension may need prolongation of ventilation. pHT may require perioperative management with chronic therapies as well as acute therapies to reduce congestion of graft
Coagulation / Transfusion
Coagulopathy monitored and indicator of graft function, viscoelastic tests
Not corrected unless bleeding or severe coagulopathy due to risks vascular thrombosis Hb target above 70 but consideration venesection if Hb > 100g/l
Immunosuppression
Should be protocoled e.g. Methylprednisolone / Azathioprine OR MMF / Tacrolimus OR Cyclosporin Variations may be institution based or patient factors e.g. Basiliximab may be given if renal dysfunction preoperatively in lieu of Calcineurin inhibitor
Infection
Routine postoperative antibiotics not necessary but will depend upon institutional protocols / intraoperative events and preoperative patient status
Postoperative IV antifungals often given in high risk cases (higher CP or MELD status)
CMV prophylaxis if CMV pos graft in CMV neg recipient
Hep B Ig and ongoing antivirals if Hep B patients
Cytotoxic precautions
Renal
Oliguria likely indicator of hypovolaemia; assess for bleeding
Consider intraabdominal hypertension
Examiner Comments:
Generally, well answered. Candidates that did poorly made generic comments about post-operative care without specific issues related to liver transplantation or lacked detail in their answers.
This is another permutation of the liver transplant SAQ, which in its other incarnations has also included some questions about why the post-transplant patient might be still unconscious after 12 hours. This time, the college asked about "specific management issues", which is ironically much less specific than the previous questions on this topic. The discussion section for the old SAQs (Question 9 from the second paper of 2015 and Question 11 from the second paper of 2012) was sufficiently detailed that it can also cover this question, and is therefore reproduced here with minimal modification. Furthermore it is worth pointing out that the college model answer to this SAQ is far better than the previous model answers.
The following management steps may be followed in the first 24 hour period:
Chapter 101 (pp. 1040) Liver transplantation by Anish Gupta, Simon Cottam and Julia Wendon
Roberts, Mark S., et al. "Survival after liver transplantation in the United States: a disease‐specific analysis of the UNOS database." Liver transplantation 10.7 (2004): 886-897.
Onaca, Nicholas N., et al. "A correlation between the pretransplantation MELD score and mortality in the first two years after liver transplantation." Liver transplantation 9.2 (2003): 117-123.
Vaid, Arjun, et al. "Molecular adsorbent recirculating system as artificial support therapy for liver failure: a meta-analysis." ASAIO Journal 58.1 (2012): 51-59.
Bañares, Rafael, et al. "Extracorporeal albumin dialysis with the molecular adsorbent recirculating system in acute‐on‐chronic liver failure: The RELIEF trial." Hepatology 57.3 (2013): 1153-1162.
Schroeder, Rebecca A., et al. "Intraoperative fluid management during orthotopic liver transplantation." Journal of cardiothoracic and vascular anesthesia 18.4 (2004): 438-441.
Kirby, R. M., et al. "Orthotopic liver transplantation: postoperative complications and their management." British journal of surgery 74.1 (1987): 3-11.
Hannaman, Michael J., and Zoltan G. Hevesi. "Anesthesia care for liver transplantation." Transplantation Reviews 25.1 (2011): 36-43.
Moreno, Rosalba, and Marina Berenguer. "Post-liver transplantation medical complications." Ann Hepatol 5.2 (2006): 77-85.
McCaughan, Geoffrey W., and Stephen R. Munn. "Liver transplantation in Australia and New Zealand." Liver Transplantation 22.6 (2016): 830-838.
A 34-year-old male has been in the ICU for almost three weeks and has undergone several laparotomies following complex abdominal trauma. He appears to have nasogastric feed emanating from his dehisced laparotomy wound and has developed a vasopressor requirement.
Give the likely diagnosis, and outline the principles of its management.
Enterocutaneous fistula (ECF) with inadequate source control
Or ECF with undrained collection
Or ECF with septic shock
Definitive surgery - may be much later
Examiner Comments:
Not well answered. Many candidates described a generic approach to intra-abdominal sepsis without considering the specific issues related to enterocutaneous fistulae.
An approach more directly relevant to enterocutaneous fistulae is described by Cohen et al (2016), Lundy & Fischer (2010) and Evenson (2016).
Cohen, Wess A., et al. "The complex surgical abdomen: What the nonsurgeon intensivist needs to know." Journal of intensive care medicine 31.4 (2016): 237-242.
Edmunds Jr, L. Henry, G. M. Williams, and Claude E. Welch. "External fistulas arising from the gastro-intestinal tract." Annals of surgery 152.3 (1960): 445.
Lundy, Jonathan B., and Josef E. Fischer. "Historical perspectives in the care of patients with enterocutaneous fistula." Clinics in colon and rectal surgery 23.3 (2010): 133.
Chapman, Richard, Robert Foran, and J. Englebert Dunphy. "Management of intestinal fistulas." The American Journal of Surgery 108.2 (1964): 157-164.
Evenson, Amy R., and Josef E. Fischer. "Current management of enterocutaneous fistula." Journal of gastrointestinal surgery 10.3 (2006): 455-464.
Gribovskaja-Rupp, Irena, and Genevieve B. Melton. "Enterocutaneous fistula: proven strategies and updates." Clinics in colon and rectal surgery 29.2 (2016): 130.
Hutchins, Robert R., et al. "Relaparotomy for suspected intraperitoneal sepsis after abdominal surgery." World journal of surgery 28.2 (2004): 137-141.
Green, Gemma, et al. "Emergency laparotomy in octogenarians: A 5-year study of morbidity and mortality." World journal of gastrointestinal surgery 5.7 (2013): 216.
Solomkin, Joseph S., et al. "Guidelines for the selection of anti-infective agents for complicated intra-abdominal infections." Clinical Infectious Diseases 37.8 (2003): 997-1005.
Montravers, P., et al. "Emergence of antibiotic-resistant bacteria in cases of peritonitis after intraabdominal surgery affects the efficacy of empirical antimicrobial therapy." Clinical Infectious Diseases 23.3 (1996): 486-494.
Latifi, R., et al. "Enterocutaneous fistulas and a hostile abdomen: reoperative surgical approaches." World journal of surgery 36.3 (2012): 516-523.
Galie, Kathryn L., and Charles B. Whitlow. "Postoperative enterocutaneous fistula: when to reoperate and how to succeed." Clinics in colon and rectal surgery 19.4 (2006): 237.
You are tasked with developing a guideline for the post-operative ICU / HDU care of patients following bariatric surgery.
a) How would you identify high risk patients who will require post-operative ICU / HDU admission? (30% marks)
b) Discuss the important post-operative issues in these patients and their management.
(70% marks)
Pre-op risk assessment
b)
Post-operative ICU Management
Altered pharmacokinetic profile in morbidly obese patients needs careful consideration
The phrase "multi-specialty involvement with a shared mental model" sounds like something the candidates should memorise and deploy alongside phrases like "therapeutic alliance" and "client-centered healthcare". It almost belongs on the glossy brochure of a private obesity clinic. Looking beyond nauseating corporate slogans, there is wisdom here: the involvement of multiple teams is required for these high-risk patients to thrive. The best single guideline statement to read for this answer was probably Mechanick et al (2013).
a)
Features of high-risk bariatric patients: In discussing this, everybody seems to quote the 2010 study by Birkmeyer et al, who assessed the features associated with hospital complications of bariatric surgery among 15,275 Michigan residents. Another highly referenced publication is the LABS consortium paper in NEJM (2009). The following factors were found to be associated with an increased risk of serious complications:
Pre-operative risk assessment for bariatric surgical patients should therefore consist of investigations which detect and (hopefully) modify some of these risks pre-operatively. Mechanick et al (2013) have an excellent preoperative checklist for this population (their Table 5), which is reproduced here with minimal modification:
Post-operative management of bariatric surgery patients
An excellent article by Thornton et al (2017) is available for the paying customers of UpToDate.
Bergeat, Damien, et al. "Postoperative outcomes of laparoscopic bariatric surgery in older obese patients: a matched case-control study." Obesity surgery 27.6 (2017): 1414-1422.
Birkmeyer, Nancy JO, et al. "Hospital complication rates with bariatric surgery in Michigan." Jama 304.4 (2010): 435-442.
Longitudinal Assessment of Bariatric Surgery (LABS) Consortium. "Perioperative safety in the longitudinal assessment of bariatric surgery." New England Journal of Medicine 361.5 (2009): 445-454.
Mechanick, Jeffrey I., et al. "Clinical practice guidelines for the perioperative nutritional, metabolic, and nonsurgical support of the bariatric surgery patient—2013 update: cosponsored by American Association of Clinical Endocrinologists, the Obesity Society, and American Society for Metabolic & Bariatric Surgery." Surgery for Obesity and Related Diseases 9.2 (2013): 159-191.
Thornton, Kevin, et al. "Bariatric surgery: Intensive care unit management of the complicated postoperative patient."
Chakravartty, Saurav, Diwakar R. Sarma, and Ameet G. Patel. "Rhabdomyolysis in bariatric surgery: a systematic review." Obesity surgery 23.8 (2013): 1333-1340.
A 45-year-old male with a history of alcohol abuse has been intubated and ventilated following an out of hospital cardiac arrest. Forty-eight hours after admission the following results were obtained:
Parameter |
Patient Value |
Adult Normal Range |
Sodium |
134 mmol/L* |
135 – 145 |
Potassium |
3.3 mmol/L* |
3.5 – 5.0 |
Chloride |
107 mmol/L* |
95 – 105 |
Bicarbonate |
19.0 mmol/L* |
22.0 – 26.0 |
Glucose |
6.7 mmol/L* |
3.5 – 6.0 |
Urea |
5.9 mmol/L |
3.0 – 8.0 |
Creatinine |
59 μmol/L |
45 – 90 |
Magnesium |
0.79 mmol/L |
0.75 – 0.95 |
Albumin |
20 g/L* |
35 – 50 |
Protein |
54 g/L* |
60 – 80 |
Total bilirubin |
82 μmol/L* |
< 26 |
Aspartate aminotransferase (AST) |
249 U/L* |
< 35 |
Alanine aminotransferase (ALT) |
41 U/L* |
< 35 |
Alkaline phosphatase (ALP) |
124 U/L* |
30 – 110 |
-Glutamyl transferase (GGT) |
481 U/L* |
< 55 |
Calcium corrected |
2.26 mmol/L |
2.12 – 2.62 |
Phosphate |
0.49 mmol/L* |
0.80 – 1.50 |
Creatinine Kinase |
114 U/L |
46 – 171 |
Lipase |
19 U/L |
< 60 |
a) Give a rationale for the results observed. (50% marks)
Mild hyponatraemia and hypokalemia may be secondary to fluid therapy or diuretic treatment. Mild acidosis may be secondary to initial ischaemic insult, or hyperchloraemic in the setting of fluid resuscitation.
Low phosphate and protein may indicate pre-existing malnutrition: risk of refeeding.
Features of liver impairment or failure with elevated total bilirubin, GGT, AST and ALT.
High AST to ALT ratio is associated with cirrhosis. rhabdomyolysis (unlikely as CK normal).
In this context both ischaemic liver damage (from out of hospital cardiac arrest) and alcoholic liver damage should be considered. However, the normal renal function may make ischaemic liver damage less likely. The high AST may reflect AMI as a precipitating factor for the arrest.
"Rationale for the results observed" appears to be a request for an unordered list of abnormalities, judging by the college answer. Thus, they are presented here in the format of abnormality(rationale)
It is unclear how this tests anything.
Outline the mechanism of action and list the indications, contraindications and complications of the TIPSS procedure (Transjugular intrahepatic portosystemic shunt).
Mechanism of action:
Percutaneous formation of a tract between hepatic vein and intrahepatic segment of portal vein. Blood is shunted away from portal circulation to systemic circulation, thereby reducing portal pressure.
Indications: (20% Marks)
Contraindications (1 mark/point to max 4 points) (30% marks)
Complications: (50% marks)
Peri-insertion:
Post insertion:
Examiners Comments:
Candidates responses lacked specific details.
In short, the rationale for a TIPS can be summarised as follows:
Indications for TIPS include:
Contraindications (from Boyer and Haskal, 2009) include
Technical complications of TIPS procedure:
Complications from portal venous shunt:
Ochs, Andreas, et al.New England Journal of Medicine 332.18 (1995): 1192-1197."The transjugular intrahepatic portosystemic stent–shunt procedure for refractory ascites."
Guevara, Mónica, et al. "Transjugular intrahepatic portosystemic shunt in hepatorenal syndrome: effects on renal function and vasoactive systems."Hepatology 28.2 (1998): 416-422.
Papatheodoridis, George V., et al. "Transjugular intrahepatic portosystemic shunt compared with endoscopic treatment for prevention of variceal rebleeding: A meta‐analysis." Hepatology 30.3 (1999): 612-622.
Azoulay, D., et al. "Transjugular intrahepatic portosystemic shunt (TIPS) for severe veno-occlusive disease of the liver following bone marrow transplantation." Bone marrow transplantation 25.9 (2000).
Ganger, Daniel R., et al. "Transjugular intrahepatic portosystemic shunt (TIPS) for Budd-Chiari syndrome or portal vein thrombosis." The American journal of gastroenterology 94.3 (1999): 603-608.
Freedman, A. M., et al. "Complications of transjugular intrahepatic portosystemic shunt: a comprehensive review." Radiographics 13.6 (1993): 1185-1210.
Rösch, J., et al. "Transjugular intrahepatic portacaval shunt an experimental work." The American Journal of Surgery 121.5 (1971): 588-592.
Rössle, Martin. "TIPS: 25years later." Journal of hepatology 59.5 (2013): 1081-1093.
Colapinto, R. F., et al. "Creation of an intrahepatic portosystemic shunt with a Grüntzig balloon catheter." Canadian Medical Association Journal 126.3 (1982): 267.
Conn, Harold O. "Transjugular Intrahepatic Portal‐systemic Shunts: The State of the Art." Hepatology 17.1 (1993): 148-158.
Gülberg, Veit, et al. "Hepatic arterial buffer response in patients with advanced cirrhosis." Hepatology 35.3 (2002): 630-634.
Boyer, Thomas D., and Ziv J. Haskal. "The role of transjugular intrahepatic portosystemic shunt (TIPS) in the management of portal hypertension: update 2009." Hepatology 51.1 (2010): 306-306.
Perarnau, Jean-Marc, et al. "Feasibility and long-term evolution of TIPS in cirrhotic patients with portal thrombosis." European journal of gastroenterology & hepatology 22.9 (2010): 1093-1098.
Orloff, Marshall J. "Emergency treatment of bleeding esophageal varices in cirrhosis." Current problems in surgery3.7 (1966): 13-28.
Unger, Lukas W., et al. "The role of TIPS in the management of liver transplant candidates." United European gastroenterology journal 5.8 (2017): 1100-1107.
DeSimone, J. A., et al. "Sustained bacteremia associated with transjugular intrahepatic portosystemic shunt (TIPS)." Clinical infectious diseases 30.2 (2000): 384-386.
Bouza, Emilio, et al. "Endotipsitis: an emerging prosthetic-related infection in patients with portal hypertension." Diagnostic microbiology and infectious disease 49.2 (2004): 77-82.
Sanyal, Arun J., and K. Rajender Reddy. "Vegetative infection of transjugular intrahepatic portosystemic shunts." Gastroenterology 115.1 (1998): 110-115.
You are asked to review a 25-year-old patient with severe diarrhoea. He is two weeks post-allogeneic hematopoietic stem cell transplant.
Causes
Drugs: promotility agents, antibiotics
Infection – multiple possible organisms
Radiation
Neutropenic enterocolitis
VHD
Outline your assessment and management
ABC
History:
Details of the indication for and type of transplant – esp. HLA matching etc.
Details of pre-transplant chemo/radiation etc.
Course and complications post-transplant
Whether engraftment has occurred
Medication history – esp. antibiotics
Details related to diarrhoea – associated with pain, blood etc,
Examination:
General physical with specific attention to Volume status
Abdominal examination – signs of peritonitis Signs of GVHD: rash, liver tenderness
Investigations:
Standard blood tests -FBC, Electrolytes, LFT’s, Cultures Stool culture including C difficile PCR
Plain AXR CT abdomen
Sigmoidoscopy +- biopsy
Discussion
Management
Fluid resuscitation – volume loss replaced by saline or balanced salt solution
Electrolyte replacement – as required but particularly potassium and magnesium
Nutrition – general enteral nutrition is satisfactory but TPN may be required where whole bowel rest is required
Antidiarrheal – loperamide can be used once infectious cause has been excluded
Anti-secretory – octreotide – particularly in early GvHD
Infection control, contact proportions
Treatment of underlying cause e.g. GVHD, C Diff.
This question is functionally quite similar to Question 11 from the first paper of 2014, except in 20144 this patient was a 65-year-old male, 18 days following BMT for multiple myeloma, and who clearly has some serious abdominal pathology.
The causes of diarrrhoea in a bone marrow transplant recipient are so vastly numerous that they require a table to manage them:
Infectious |
Non-Infectious |
Viruses
Bacteria:
Parasites
Fungi
|
Immunosuppressant therapy
Consequences of BMT
ICU therapy
|
Assessment:
Management:
UpToDate have a good article on acute diarrhoea for the paying customer.
Kelly, T. W. J., M. R. Patrick, and K. M. Hillman. "Study of diarrhea in critically ill patients." Critical care medicine 11.1 (1983): 7-9.
Wiesen, Patricia, Andre Van Gossum, and Jean-Charles Preiser. "Diarrhoea in the critically ill." Current opinion in critical care 12.2 (2006): 149-154.
Ferrie, Suzie, and Vivienne East. "Managing diarrhoea in intensive care."Australian Critical Care 20.1 (2007): 7-13.
Pawlowski, Sean W., Cirle Alcantara Warren, and Richard Guerrant. "Diagnosis and treatment of acute or persistent diarrhea." Gastroenterology 136.6 (2009): 1874-1886.
Schiller, Lawrence R. "Diarrhea." Medical Clinics of North America 84.5 (2000): 1259-1274.
Timothy A. Woods. "Diarrhea." Chapter 88 in: Clinical Methods: The History, Physical, and Laboratory Examinations. 3rd edition. Boston: Butterworths; 1990.
Guerrant, Richard L., et al. "Practice guidelines for the management of infectious diarrhea." Clinical infectious diseases 32.3 (2001): 331-351.
Cox, George J., et al. "Etiology and outcome of diarrhea after marrow transplantation: a prospective study." Gastroenterology 107.5 (1994): 1398-1407.
Hamdeh, Shadi, et al. "Clinical approach to diarrheal disorders in allogeneic hematopoietic stem cell transplant recipients." World Journal of Hematology 5.1 (2016): 23-30.
A 60-year-old patient with regular heavy alcohol intake is admitted to ICU in a stuporous state after a two week history of difficulty in walking and repeated falls. On examination, his Glasgow Coma Score is E2M4V3, with bilateral nystagmus and limited outward movement of both eyes on turning his head. There is no neck stiffness, asterixis or focal neurological deficit. A CT scan of the brain shows generalised cerebral atrophy.
a) What is the diagnosis? (15% marks)
b) What is the specific treatment for this condition? (10% marks)
1. Wernicke’s Encephalopathy
2. IV Thiamine in high doses (500mg thrice a day x 2 days; 250mg daily thereafter) followed by IV Glucose
Those trainees who are wondering what the scientific definition of "stuporous" is will be enouraged to learn that there isn't one. Merriam-Webster defines it as
"a condition of greatly dulled or completely suspended sense or sensibility; specifically : a chiefly mental condition marked by absence of spontaneous movement, greatly diminished responsiveness to stimulation, and usually impaired consciousness".
One would think that this sounds very colloquial and 19th century (where a young lady might have been accused of having greatly dulled or completely suspended sense or sensibility for rejecting a wealthy suitor). A slightly more modern and medical-sounding definition can be found in the 2007 edition of Plum & Posner's Diagnosis of stupor and coma (p.7):
"Stupor, from the Latin ‘‘to be stunned,’’ is a condition of deep sleep or similar behavioral unresponsiveness from which the subject can be aroused only with vigorous and continuous stimulation. Even when maximally aroused, the level of cognitive function may be impaired. Such patients can be differentiated from those with psychiatric impairment, such as catatonia or severe depression, because they can be aroused by vigorous stimulation to respond to simple stimuli."
Anyway. This is clearly Wernicke's encephalopathy. Recall its features, of which this patient has several:
The specific treatment is IV thiamine. There is a disagreement as to how much is actually enough (The college answer to Question 13.3 from the second paper of 2013 suggests 100mg IV daily is a big enough dose, whereas Cook et al (1998) recommended 1g daily).
Johnson, J. "Stupor: a review of 25 cases." Acta Psychiatrica Scandinavica 70.4 (1984): 370-377.
Flynn, Alexandra, et al. "Wernicke’s Encephalopathy: Increasing Clinician Awareness of This Serious, Enigmatic, Yet Treatable Disease." The primary care companion for CNS disorders 17.3 (2015).
Thomson, Allan D., and E. Jane Marshall. "The natural history and pathophysiology of Wernicke's encephalopathy and Korsakoff's psychosis." Alcohol and Alcoholism 41.2 (2006): 151-158.
Gussow, Leon. "Myths of toxicology: thiamine before dextrose." Emergency Medicine News 29.4 (2007): 3-11.
Isenberg-Grzeda, Elie, Haley E. Kutner, and Stephen E. Nicolson. "Wernicke-Korsakoff-syndrome: under-recognized and under-treated." Psychosomatics 53.6 (2012): 507-516.\
Watson, A. J. S., et al. "Acute Wernickes encephalopathy precipitated by glucose loading." Irish journal of medical science 150.1 (1981): 301-303.
Kissoon, Niranjan. "Thiamine before glucose to prevent Wernicke encephalopathy: examining the conventional wisdom." JAMA 279.8 (1998): 583.
Day, Ed, et al. "Thiamine for prevention and treatment of Wernicke‐Korsakoff Syndrome in people who abuse alcohol." The Cochrane Library (2013).
Ambrose, Margaret L., Stephen C. Bowden, and Greg Whelan. "Thiamin Treatment and Working Memory Function of Alcohol‐Dependent People: Preliminary Findings." Alcoholism: Clinical and Experimental Research 25.1 (2001): 112-116.
Cook, Christopher CH, Phillip M. Hallwood, and Allan D. Thomson. "B Vitamin deficiency and neuropsychiatric syndromes in alcohol misuse." Alcohol and Alcoholism 33.4 (1998): 317-336.
A 40-year-old patient with a background of alcohol abuse presents with a history of 8 days of diarrhoea and vomiting.
The following results are obtained:
Parameter |
Patient Value |
Adult Normal Range |
Sodium |
116 mmol/L* |
137 – 146 |
Potassium |
2.9 mmol/L* |
3.5 – 5.0 |
Chloride |
67 mmol/L* |
95 – 110 |
Bicarbonate |
14 mmol/L* |
24 – 31 |
Urea |
2.9 mmol/L* |
3.0 – 8.5 |
Creatinine |
46 µmol/L* |
60 – 120 |
Glucose |
6.8 mmol/L |
3.0 – 7.8 |
Osmolality |
254 mOsm/kg* |
274 – 295 |
Phosphate |
0.6 mmol/L* |
0.7 – 1.4 |
Magnesium |
0.7 mmol/L |
0.7 – 1.05 |
Calcium corrected |
2.3 mmol/L |
2.1 – 2.6 |
Albumin |
44 g/L |
36 – 52 |
Bilirubin |
13 µmol/L |
0 – 18 |
Aspartate transferase |
80 U/L* |
0 – 30 |
Alanine transferase |
67 U/L* |
0 – 30 |
Alkaline phosphatase |
148 U/L* |
30 – 100 |
g-Glutamyl transferase |
480 U/L* |
0 – 35 |
a) What is the acid-base disturbance in this patient? (20% marks)
b) What are the likely causes in this context? (30% marks)
a)
Metabolic acidosis- Anion gap= 116-(67+14) = 35 Delta ratio= 23/10=2.3
HAGMA with metabolic alkalosis OR Increased SID
b)
Metabolic alkalosis – due to vomiting- Acid loss and contraction alkalosis. HAGMA-
Lactic acidosis from hypovolaemia or bowel obstruction/sepsis, Ketoacidosis from starvation/alcohol.
In some sort of a structured fashion:
Causes of HAGMA appropriate to this scenario include:
The metabolic alkalosis can be attributed to the vomiting and diarrhoea, and more directly to the aldosterone excess which develops in states of volume depletion.
With regards to open surgery for carcinoma of the head of the pancreas, list the anastomoses formed during a Whipples procedure. (30% marks)
List the complications of this procedure that are of relevance to its ICU management. (70% marks)
(A list of anastomoses was acceptable; the diagram is provided for clarity).
b)
List complications specific to this procedure of relevance to the Intensive Care management of patients who have had this procedure. (7 marks)
Specific early surgical complications (4 marks) Primary haemorrhage
Pancreatic fistula Pancreatitis
Bile leak
GI anastomotic failure Portal or SMA thrombosis Diabetes Mellitus
Gastric outlet obstruction Late complications
Anastomotic stricture Delayed gastric emptying Pancreatic fistula
Gastric fistula
Malabsorption and electrolyte abnormalities secondary to the above
Examiners Comments:
There was poor knowledge of the anatomy of a procedure that is commonly encountered in intensive care.
The sites of anastomosis:
This image is from thefreedictionary.com.
As one might imagine of a 6-8 hour procedure which ends with three anastomoses in the abdomen, "the complications of this procedure that are of relevance to its ICU management" are numerous, and making a coherent answer to this sort of question really relies on the candidate's abilities to categorise them quickly.
Surgical complications
Airway-related concerns
Respiratory complications
Vascular complications
Pain
Electrolyte derangement
Fluid balance
Nutritional problems
Intra-abdominal collections (12.2%)
Thromboembolic disease
Schnelldorfer, Thomas, and Michael G. Sarr. "Alessandro Codivilla and the first pancreatoduodenectomy." Archives of Surgery 144.12 (2009): 1179-1184.
Kausch, W. "Das Carcinom der Papilla duodeni und seine radikale Entfernung." Beitr Klin Chir. 78 (1912): 439-486.
Are, Chandrakanth, Mashaal Dhir, and Lavanya Ravipati. "History of pancreaticoduodenectomy: early misconceptions, initial milestones and the pioneers." Hpb 13.6 (2011): 377-384.
Whipple, A. O. "Observations on radical surgery for lesions of the pancreas." Surg Gynecol Obstet 82 (1946): 623-631.
Whipple, Allen O., William Barclay Parsons, and Clinton R. Mullins. "Treatment of carcinoma of the ampulla of Vater." Annals of surgery 102.4 (1935): 763.
Butturini, Giovanni, et al. "Complications after pancreaticoduodenectomy: the problem of current definitions." Journal of hepato-biliary-pancreatic surgery 13.3 (2006): 207-211.
Karim, Sherko Abdullah Molah, et al. "The outcomes and complications of pancreaticoduodenectomy (Whipple procedure): Cross sectional study." International Journal of Surgery 52 (2018): 383-387.
McEvoy, S. H., et al. "Pancreaticoduodenectomy: expected post-operative anatomy and complications." The British journal of radiology 87.1041 (2014): 20140050.
Keim, Volker, et al. "Postoperative care following pancreatic surgery: surveillance and treatment." Deutsches Ärzteblatt International 106.48 (2009): 789.
A 55-year-old patient is admitted with an exacerbation of chronic liver disease. Results of an ascitic tap and serum results are listed below:
ASCITIC FLUID |
|
Parameter |
Patient Value |
Appearance |
Clear Yellow |
pH |
micro-clots present, no value obtained |
Red Cell Count |
0 erythrocytes/µL |
White Cell Count |
378 leukocytes/µL |
Ascitic Fluid Protein |
25 g/L |
Ascitic Fluid Albumin |
18 g/L |
Ascitic Fluid Lactic Acid Dehydrogenase (LOH) |
480 U/L |
Gram Stain |
no organisms seen |
SERUM |
|||||
Parameter |
Patient Value |
Adult Normal Range |
|||
Serum |
Protein |
32 g/L* |
60-80 |
||
Serum |
Albumin |
23 g/L* |
35-50 |
||
Serum Lactic Acid Dehydrogenase (LOH) |
320 U/L* |
120- 250 |
a) List four possible underlying diagnoses. (20% marks)
Spontaneous bacterial peritonitis
Perforated viscus
Cirrhosis
Malignancy
TB
Carrying on with theme for Question 5, this is another middle-aged gentleman with liver disease. This time, the question really is, "what does this ascitic tap mean". The history offered with the results ("exacerbation of liver disease"?) is so uninformative it may as well not exist.
So: using the 2009 article by Koulaouzidis and Such & Runyon (1998) as the main sources, one can concoct an unordered list which describes the reported findings, and their possible meaning.
So, in summary, what could this be?
Ward, Patrick CJ. "Interpretation of ascitic fluid data." Postgraduate medicine 71.2 (1982): 171-178.
Tarn, A. C., and R. Lapworth. "Biochemical analysis of ascitic (peritoneal) fluid: what should we measure?." Annals of Clinical Biochemistry 47.5 (2010): 397-407.
Bar-Meir, Simon, Emanuel Lerner, and Harold O. Conn. "Analysis of ascitic fluid in cirrhosis." Digestive diseases and sciences 24.2 (1979): 136-144.
Boyer, Thomas D., Arthur M. Kahn, and Telfer B. Reynolds. "Diagnostic value of ascitic fluid lactic dehydrogenase, protein, and WBC levels." Archives of internal medicine 138.7 (1978): 1103-1105.
Runyon, Bruce A., John C. Hoefs, and Timothy R. Morgan. "Ascitic fluid analysis in malignancy‐related ascites." Hepatology 8.5 (1988): 1104-1109.
Al-Mandeel, Hazem, and Abeer Qassem. "Urinary ascites secondary to delayed diagnosis of laparoscopic bladder injury." Journal of minimal access surgery 6.2 (2010): 50.
Develing, L., J. F. Hamming, and B. Speelberg. "[Chylous ascites following surgical repair of a ruptured abdominal aortic aneurysm]." Nederlands tijdschrift voor geneeskunde 147.31 (2003): 1513-1516.
Frank, Denis J., et al. "Traumatic rupture of the gallbladder with massive biliary ascites." JAMA 240.3 (1978): 252-253.
Cameron, JOHN L., et al. "Internal pancreatic fistulas: pancreatic ascites and pleural effusions." Annals of surgery 184.5 (1976): 587.
Berner, C., et al. "Diagnosis of ascites." British Medical Journal 282 (1981): 1499.
BERNER, CHARLES, et al. "Diagnostic probabilities in patients with conspicuous ascites." Archives of internal medicine 113.5 (1964): 687-690.
Aminiahidashti, Hamed, et al. "Diagnostic Accuracy of Ascites Fluid Gross Appearance in Detection of Spontaneous Bacterial Peritonitis." Emergency 2.3 (2014): 138.
Gitlin, Norman, John L. Stauffer, and Ronald C. Silvestri. "The pH of ascitic fluid in the diagnosis of spontaneous bacterial peritonitis in alcoholic cirrhosis." Hepatology 2.4 (1982): 408S-411S.
Urrunaga, Nathalie H., et al. "Hemorrhagic ascites. Clinical presentation and outcomes in patients with cirrhosis." Journal of hepatology 58.6 (2013): 1113-1118.
Pare, Pierre, Jean Talbot, and John C. Hoefs. "Serum-ascites albumin concentration gradient: a physiologic approach to the differential diagnosis of ascites." Gastroenterology 85.2 (1983): 240-244.
Boyer, Thomas D., Arthur M. Kahn, and Telfer B. Reynolds. "Diagnostic value of ascitic fluid lactic dehydrogenase, protein, and WBC levels." Archives of internal medicine 138.7 (1978): 1103-1105.
Banerjee, Mithu, et al. "Biomarkers of malignant ascites—a myth or reality." Medical Journal Armed Forces India 67.2 (2011): 108-112.
Ekpe, E. E. L., and A. J. Omotoso. "The Relevance of Ascitic Lactate Dehydrogenase (LDH) and Serum Ascites Albumin Gradient (SAAG) in the Differential Diagnosis of Ascites among Patients in a Nigerian Hospital." Journal of Advances in Medicine and Medical Research (2015): 211-219.
Pattinson, H. A., et al. "Clotting and fibrinolytic activities in peritoneal fluid." BJOG: An International Journal of Obstetrics & Gynaecology 88.2 (1981): 160-166.
Critically evaluate the role of proton pump inhibitors to prevent upper gastrointestinal bleeding in ICU patients.
Rationale
Upper GI bleeding can occur due to stress ulceration in critically ill patients; risk of clinically significant bleeding estimated at around 1.5% in ventilated patients taking stress ulcer prophylaxis, historically up to 15% in those without prophylaxis.
Major risk factors for GI bleeding appear to be duration of mechanical ventilation and presence of a coagulopathy, also use of steroids, past history of peptic ulcer disease [Cook NEJM 1994]
Enteral nutrition may be preventative [Marik Crit Care Med 2010]
Many studies have shown a reduction in GI bleeding with the use of prophylaxis
PPIs are very effective at treating stress ulcer-related bleeding and are the most potent medications available to prevent GI bleeding in ICU patients [Barkun AN et al Gastroenterol 2012 Apr; 107(4)] Cook et al 2013, Int Care Med 2018).
Disadvantages
Side effects of use of PPI may include increase risk of VAP, C. Difficile infection, acute interstitial nephritis, and cost (included unintended long-term use). They may have an immunosuppressive effect.
Evidence
SUP-ICU [NEJM 2018] demonstrated that PPI use compared to placebo resulted in a reduced rate of clinically important GI bleeding (2.5% vs 4.2%], NNT = 59. No difference in mortality.
PEPTIC (JAMA 2020) demonstrated that in ventilated ICU patients, PPIs were more effective at reducing GI bleeding than H2RBs. No effect on mortality, ICU LOS or C Difficile infection rate. The study had a high crossover rate.
In a subset of cardiac surgical patients, the GI bleeding rate was very low, and mortality was increased with allocation to PPI group.
REVISE trial currently underway will provide an update on the beneficial effect of PPI compared to placebo for stress ulcer prophylaxis in ICU patients.
Overall, there is a clear need to define high-risk critically ill patient sub-group that is likely to benefit from stress ulcer prophylaxis, accounting for those that receive enteral nutrition. In light of PEPTIC, it is unlikely that PPIs offer a mortality benefit over H2RBs.
Summary: (candidates should justify their own practice; there is not currently a clear “correct” answer for this and so this serves as an example only):
In my practice, I would only use stress ulcer prophylaxis in ICU patients who are at high risk of GI bleeding (mechanically ventilated >48 hours and either: coagulopathy; shock/MODS/high illness severity; or high dose corticosteroids)
I would use H2RB as my standard prophylaxis medication
I would use PPI as treatment for any patients with signs of GI bleeding
I would assess the ongoing need for prophylaxis daily and cease when patient is no longer high risk, including when enteral nutrition is commenced.
I would routinely cease stress ulcer prophylaxis prior to discharge to the wards
Good answers contained the following points:
The rationale for using them. Advantages (cheap, widely available) Potential disadvantages
A summary of the evidence. The key points would be that they do appear to lower the incidence of GI bleeding, but do not seem to reduce mortality (and some suggestion that mortality might be increased in particular populations). A grasp of what the evidence suggests was sufficient, although detailed knowledge of recent studies was awarded marks.
A summary statement.
Rationale
Advantages
Disadvantages
Evidence for the routine use of ulcer prophylaxis
Evidence to support one drug class over another
Summary
PPIs are indicated in at-risk patient in ICU who are intolerant of enteral feeding, and who are otherwise at risk of gastrointestinal bleeding. Further research is required to discriminated between different classes of drugs in terms of efficacy, and to identify the at-risk population.
Oh's Intensive Care manual: Chapter 42 (pp. 487) Acute gastrointestinal bleeding by Joseph JY Sung
Cook, Deborah J., et al. "Risk factors for gastrointestinal bleeding in critically ill patients." New England Journal of Medicine 330.6 (1994): 377-381.
Marik, Paul E., et al. "Stress ulcer prophylaxis in the new millennium: a systematic review and meta-analysis." Crit Care Med 38.11 (2010): 2222-2228.
Krag, Mette, et al. "Stress ulcer prophylaxis versus placebo or no prophylaxis in critically ill patients." Intensive care medicine 40.1 (2014): 11-22.
Madsen, Kristian Rørbæk, et al. "Guideline for Stress Ulcer Prophylaxis in the Intensive Care Unit." Danish medical journal 61.3 (2014): 1-4.
Plummer, Mark P., Annika Reintam Blaser, and Adam M. Deane. "Stress ulceration: prevalence, pathology and association with adverse outcomes."Critical Care 18.2 (2014): 213.
Yearsley, K. A., et al. "Proton pump inhibitor therapy is a risk factor for Clostridium difficile‐associated diarrhoea." Alimentary pharmacology & therapeutics 24.4 (2006): 613-619.
Steinberg, Kenneth P. "Stress-related mucosal disease in the critically ill patient: risk factors and strategies to prevent stress-related bleeding in the intensive care unit." Critical care medicine 30.6 (2002): S362-S364.
Buendgens, Lukas, Alexander Koch, and Frank Tacke. "Prevention of stress-related ulcer bleeding at the intensive care unit: Risks and benefits of stress ulcer prophylaxis." World journal of critical care medicine 5.1 (2016): 57.
Gonzalez, Edgar R., and Anthony R. Morkunas. "Prophylaxis of stress ulcers: Antacid titration vs. histamine2-receptor blockade." Drug intelligence & clinical pharmacy 19.11 (1985): 807-811.
Cook, Deborah, et al. "A comparison of sucralfate and ranitidine for the prevention of upper gastrointestinal bleeding in patients requiring mechanical ventilation." New England Journal of Medicine 338.12 (1998): 791-797.
Oh's Intensive Care manual: Chapter 42 (pp. 487) Acute gastrointestinal bleeding by Joseph JY Sung
Cook, Deborah J., et al. "Risk factors for gastrointestinal bleeding in critically ill patients." New England Journal of Medicine 330.6 (1994): 377-381.
Marik, Paul E., et al. "Stress ulcer prophylaxis in the new millennium: a systematic review and meta-analysis." Crit Care Med 38.11 (2010): 2222-2228.
Krag, Mette, et al. "Stress ulcer prophylaxis versus placebo or no prophylaxis in critically ill patients." Intensive care medicine 40.1 (2014): 11-22.
Madsen, Kristian Rørbæk, et al. "Guideline for Stress Ulcer Prophylaxis in the Intensive Care Unit." Danish medical journal 61.3 (2014): 1-4.
Plummer, Mark P., Annika Reintam Blaser, and Adam M. Deane. "Stress ulceration: prevalence, pathology and association with adverse outcomes."Critical Care 18.2 (2014): 213.
Yearsley, K. A., et al. "Proton pump inhibitor therapy is a risk factor for Clostridium difficile‐associated diarrhoea." Alimentary pharmacology & therapeutics 24.4 (2006): 613-619.
Steinberg, Kenneth P. "Stress-related mucosal disease in the critically ill patient: risk factors and strategies to prevent stress-related bleeding in the intensive care unit." Critical care medicine 30.6 (2002): S362-S364.
Buendgens, Lukas, Alexander Koch, and Frank Tacke. "Prevention of stress-related ulcer bleeding at the intensive care unit: Risks and benefits of stress ulcer prophylaxis." World journal of critical care medicine 5.1 (2016): 57.
Gonzalez, Edgar R., and Anthony R. Morkunas. "Prophylaxis of stress ulcers: Antacid titration vs. histamine2-receptor blockade." Drug intelligence & clinical pharmacy 19.11 (1985): 807-811.
Cook, Deborah, et al. "A comparison of sucralfate and ranitidine for the prevention of upper gastrointestinal bleeding in patients requiring mechanical ventilation." New England Journal of Medicine 338.12 (1998): 791-797.
MacLaren, Robert, Catherine L. Jarvis, and Douglas N. Fish. "Use of enteral nutrition for stress ulcer prophylaxis." Annals of Pharmacotherapy 35.12 (2001): 1614-1623.
El-Kersh, Karim, et al. "Enteral nutrition as stress ulcer prophylaxis in critically ill patients: A randomized controlled exploratory study." Journal of critical care 43 (2018): 108-113.
Alhazzani, Waleed, et al. "Withholding pantoprazole for stress ulcer prophylaxis in critically ill patients: a pilot randomized clinical trial and meta-analysis." Critical care medicine 45.7 (2017): 1121-1129.
Lucas, Charles E., et al. "Natural history and surgical dilemma of stress gastric bleeding." Archives of surgery 102.4 (1971): 266-273.
Hastings, Paul R., et al. "Antacid titration in the prevention of acute gastrointestinal bleeding: A controlled, randomized trial in 100 critically ill patients." New England Journal of Medicine298.19 (1978): 1041-1045.
Krag, Mette, et al. "Stress ulcer prophylaxis with a proton pump inhibitor versus placebo in critically ill patients (SUP-ICU trial): study protocol for a randomised controlled trial." Trials17.1 (2016): 205.
Krag, Mette, et al. "Pantoprazole in patients at risk for gastrointestinal bleeding in the ICU." New England Journal of Medicine 379.23 (2018): 2199-2208.
Using the headings of history, examination, biochemical findings, haematological findings, imaging and biopsy, compare and contrast the clinical features and investigation findings of Acute Fulminant Hepatic Failure with Decompensated Chronic Liver Disease
Acute Fulminant |
Decompensated Chronic |
|
History |
Acute, toxic ingestion (examples) Viral Infections, Ischaemia Post-surgery |
Chronic Liver disease Encephalopathy GI Bleeding |
Examination |
Jaundice |
Jaundice |
Coma |
Coma |
|
Intracranial hypertension common |
Intracranial hypertension rare |
|
No signs chronic liver disease |
Signs chronic liver disease |
|
No ascites |
Ascites/oedema |
|
No portal hypertension |
Signs of Portal hypertension |
|
Biochemistry |
Hypoglycaemia common |
Hypoglycaemia rare |
Hyponatraemia rare |
Hyponatraemia common |
|
Severe acidosis |
Mild acidosis |
|
High Bilirubin |
High Bilirubin |
|
LFT’s severely deranged |
LFT’s mildly deranged |
|
Renal failure common |
Renal failure less common |
|
Haematology |
Platelets usually normal |
Platelets usually low |
INR very high |
INR mildly raised |
|
Fibrinogen low |
Fibrinogen normal |
|
Imaging |
||
Head |
Cerebral oedema |
No cerebral oedema |
Abdomen |
Normal – may show swollen liver or vascular cause of fulminant failure |
Small fibrotic/cirrhotic liver, splenomegaly, varices. |
Biopsy |
Necrosis |
Fibrosis |
This question is well-formed and gives an accurate impression of exactly what the college wanted to from this comparison of Inappropriately Capitalised Conditions.
Domain | Acute fulminant hepatic failure | Decompensated chronic liver disease |
History |
|
|
Examination |
|
|
Biochemistry |
|
|
Haematology |
|
|
Imaging |
|
|
Biopsy |
|
|
CT brain |
|
|
Credit was given to any additional correct answers under the relevant headings.
Oh's Manual: Chapter 44 (pp. 501) Liver failure by Christopher Willars and Julia Wendon
Kim, Tae Yeob, and Dong Joon Kim. "Acute-on-chronic liver failure." Clinical and molecular hepatology 19.4 (2013): 349.
Bernal, William, et al. "Acute liver failure." The Lancet 376.9736 (2010): 190-201.
Outline the pathophysiology, diagnosis, and treatment of mesenteric ischaemia in the critically ill patient.
Not available.
This question is functionally identical to Question 12 from the first paper of 2017, except this time the examiners appended "in the critically ill patient", presumably because last time all the trainees spent too much time discussing mesenteric ischaemia among the well patients out in the community.
Pathophysiology
History
Examination
Biochemistry
Imaging
Specific management
Supportive management
Acosta, Stefan, and Martin Björck. "Modern treatment of acute mesenteric ischaemia." British Journal of Surgery 101.1 (2014).
Schofield, Nick, et al. "Acute mesenteric ischaemia." Journal of the Intensive Care Society 15.3 (2014): 226-230.
Clair, Daniel G., and Jocelyn M. Beach. "Mesenteric ischemia." New England Journal of Medicine 374.10 (2016): 959-968. (pdf)
Pecoraro, Felice, et al. "Chronic mesenteric ischemia: critical review and guidelines for management." Annals of vascular surgery 27.1 (2013): 113-122.
Outline the mechanism of action and list the indications, contraindications, and complications of the TIPSS procedure (Transjugular intrahepatic portosystemic shunt).
Not available.
Mechanism of TIPS:
Indications:
Contraindications for portal venous shunt (Boyer and Haskal, 2009)
Technical complications:
Complications from portal venous shunt:
Ochs, Andreas, et al.New England Journal of Medicine 332.18 (1995): 1192-1197."The transjugular intrahepatic portosystemic stent–shunt procedure for refractory ascites."
Guevara, Mónica, et al. "Transjugular intrahepatic portosystemic shunt in hepatorenal syndrome: effects on renal function and vasoactive systems."Hepatology 28.2 (1998): 416-422.
Papatheodoridis, George V., et al. "Transjugular intrahepatic portosystemic shunt compared with endoscopic treatment for prevention of variceal rebleeding: A meta‐analysis." Hepatology 30.3 (1999): 612-622.
Azoulay, D., et al. "Transjugular intrahepatic portosystemic shunt (TIPS) for severe veno-occlusive disease of the liver following bone marrow transplantation." Bone marrow transplantation 25.9 (2000).
Ganger, Daniel R., et al. "Transjugular intrahepatic portosystemic shunt (TIPS) for Budd-Chiari syndrome or portal vein thrombosis." The American journal of gastroenterology 94.3 (1999): 603-608.
Freedman, A. M., et al. "Complications of transjugular intrahepatic portosystemic shunt: a comprehensive review." Radiographics 13.6 (1993): 1185-1210.
Rösch, J., et al. "Transjugular intrahepatic portacaval shunt an experimental work." The American Journal of Surgery 121.5 (1971): 588-592.
Rössle, Martin. "TIPS: 25years later." Journal of hepatology 59.5 (2013): 1081-1093.
Colapinto, R. F., et al. "Creation of an intrahepatic portosystemic shunt with a Grüntzig balloon catheter." Canadian Medical Association Journal 126.3 (1982): 267.
Conn, Harold O. "Transjugular Intrahepatic Portal‐systemic Shunts: The State of the Art." Hepatology 17.1 (1993): 148-158.
Gülberg, Veit, et al. "Hepatic arterial buffer response in patients with advanced cirrhosis." Hepatology 35.3 (2002): 630-634.
Boyer, Thomas D., and Ziv J. Haskal. "The role of transjugular intrahepatic portosystemic shunt (TIPS) in the management of portal hypertension: update 2009." Hepatology 51.1 (2010): 306-306.
Perarnau, Jean-Marc, et al. "Feasibility and long-term evolution of TIPS in cirrhotic patients with portal thrombosis." European journal of gastroenterology & hepatology 22.9 (2010): 1093-1098.
Orloff, Marshall J. "Emergency treatment of bleeding esophageal varices in cirrhosis." Current problems in surgery3.7 (1966): 13-28.
Unger, Lukas W., et al. "The role of TIPS in the management of liver transplant candidates." United European gastroenterology journal 5.8 (2017): 1100-1107.
DeSimone, J. A., et al. "Sustained bacteremia associated with transjugular intrahepatic portosystemic shunt (TIPS)." Clinical infectious diseases 30.2 (2000): 384-386.
Bouza, Emilio, et al. "Endotipsitis: an emerging prosthetic-related infection in patients with portal hypertension." Diagnostic microbiology and infectious disease 49.2 (2004): 77-82.
Sanyal, Arun J., and K. Rajender Reddy. "Vegetative infection of transjugular intrahepatic portosystemic shunts." Gastroenterology 115.1 (1998): 110-115.
Discuss the management of Enterocutaneous fistulae (ECF) in the critically ill patient.
Not available.
An approach directly relevant to enterocutaneous fistulae is described by Cohen et al (2016), Lundy & Fischer (2010) and Evenson (2016).
Cohen, Wess A., et al. "The complex surgical abdomen: What the nonsurgeon intensivist needs to know." Journal of intensive care medicine 31.4 (2016): 237-242.
Edmunds Jr, L. Henry, G. M. Williams, and Claude E. Welch. "External fistulas arising from the gastro-intestinal tract." Annals of surgery 152.3 (1960): 445.
Lundy, Jonathan B., and Josef E. Fischer. "Historical perspectives in the care of patients with enterocutaneous fistula." Clinics in colon and rectal surgery 23.3 (2010): 133.
Chapman, Richard, Robert Foran, and J. Englebert Dunphy. "Management of intestinal fistulas." The American Journal of Surgery 108.2 (1964): 157-164.
Evenson, Amy R., and Josef E. Fischer. "Current management of enterocutaneous fistula." Journal of gastrointestinal surgery 10.3 (2006): 455-464.
Gribovskaja-Rupp, Irena, and Genevieve B. Melton. "Enterocutaneous fistula: proven strategies and updates." Clinics in colon and rectal surgery 29.2 (2016): 130.
Hutchins, Robert R., et al. "Relaparotomy for suspected intraperitoneal sepsis after abdominal surgery." World journal of surgery 28.2 (2004): 137-141.
Green, Gemma, et al. "Emergency laparotomy in octogenarians: A 5-year study of morbidity and mortality." World journal of gastrointestinal surgery 5.7 (2013): 216.
Solomkin, Joseph S., et al. "Guidelines for the selection of anti-infective agents for complicated intra-abdominal infections." Clinical Infectious Diseases 37.8 (2003): 997-1005.
Montravers, P., et al. "Emergence of antibiotic-resistant bacteria in cases of peritonitis after intraabdominal surgery affects the efficacy of empirical antimicrobial therapy." Clinical Infectious Diseases 23.3 (1996): 486-494.
Latifi, R., et al. "Enterocutaneous fistulas and a hostile abdomen: reoperative surgical approaches." World journal of surgery 36.3 (2012): 516-523.
Galie, Kathryn L., and Charles B. Whitlow. "Postoperative enterocutaneous fistula: when to reoperate and how to succeed." Clinics in colon and rectal surgery 19.4 (2006): 237.
A 45-year-old male with a background of chronic liver disease is admitted to the Emergency Department with massive haematemesis secondary to a variceal bleed. He is managed with endoscopy and sclerotherapy.
a) List the clinical indicators for risk of re-bleeding from the gastric varices. (20% marks)
b) List the pharmacological agents that may reduce the risk of a re-bleed. (20% marks)
c) Briefly discuss the haemoglobin transfusion trigger you will use in the clinical management of this patient (20% marks)
d) List, in order of priority, four specific non-pharmacological options for controlling variceal re-bleed AND, where appropriate, the relative advantages and disadvantages of these.
(40% marks)
Not available.
This question is very similar to Question 1 from the first paper of 2017.
a) List the clinical indicators for risk of re-bleeding from the gastric varices.
From Augustine et al (2010), there are several features found to be strongly associated with "five-day failure", a composite endpoint consisting of re-bleeding and five-day mortality. All of these have been combined into this list:
b) List the pharmacological agents that may reduce the risk of a re-bleed.
c) Briefly discuss the haemoglobin transfusion trigger you will use in the clinical management of this patient
A haemoglobin transfusion trigger is not usually the way one decides to give a transfusion, as the decision should be based on a series of clinical assessments and weighing the risks andbenefits of blood transfusion. There is not a lot of data to describe numerical transfusion thresholds in this population. One study which could help is Rockey (2014), where the authors reported better outcomes with a transfusion target of 70g/L vs. 90g/L.
d) List, in order of priority, four specific non-pharmacological options for controlling variceal re-bleed AND, where appropriate, the relative advantages and disadvantages of these.
Repeat endoscopy |
|
|
Balloon tamponade |
|
|
TIPS |
|
|
Surgical control |
|
|
Balloon-occluded retrograde transverse obliteration (BRTO) |
|
|
Rockey, Don C. "To transfuse or not to transfuse in upper gastrointestinal hemorrhage? That is the question." Hepatology 60.1 (2014): 422-424.
Oh's Intensive Care manual: Chapter 42 (pp. 487) Acute gastrointestinal bleeding by Joseph JY Sung
arcia-Tsao, Guadalupe, and Jaime Bosch. "Management of varices and variceal hemorrhage in cirrhosis." New England Journal of Medicine 362.9 (2010): 823-832.
García-Pagán, Juan Carlos, et al. "Early use of TIPS in patients with cirrhosis and variceal bleeding." New England Journal of Medicine 362.25 (2010): 2370-2379.
Vlavianos, P., et al. "Balloon tamponade in variceal bleeding: use and misuse."BMJ: British Medical Journal 298.6681 (1989): 1158.
Reverter, Enric, and Juan Carlos García‐Pagán. "Management of an acute variceal bleeding episode." Clinical Liver Disease 1.5 (2012): 151-154.
Ioannou, G. N., J. Doust, and D. C. Rockey. "Terlipressin in acute oesophageal variceal haemorrhage." Alimentary pharmacology & therapeutics 17.1 (2003): 53-64.
Corley, Douglas A., et al. "Octreotide for acute esophageal variceal bleeding: a meta-analysis." Gastroenterology 120.4 (2001): 946-954.
Reiberger, Thomas, et al. "Carvedilol for primary prophylaxis of variceal bleeding in cirrhotic patients with haemodynamic non-response to propranolol." Gut62.11 (2013): 1634-1641.
Hou, Ming‐Chih, et al. "Antibiotic prophylaxis after endoscopic therapy prevents rebleeding in acute variceal hemorrhage: a randomized trial." Hepatology 39.3 (2004): 746-753.
Augustin, Salvador, Antonio González, and Joan Genescà. "Acute esophageal variceal bleeding: Current strategies and new perspectives." World J Hepatol 2.7 (2010): 261-274.
Chen, Ping-Hsien, et al. "Delayed endoscopy increases re-bleeding and mortality in patients with hematemesis and active esophageal variceal bleeding: a cohort study." Journal of hepatology 57.6 (2012): 1207-1213.
Kleber, Gerhard, et al. "Prediction of variceal hemorrhage in cirrhosis: a prospective follow-up study." Gastroenterology 100.5 (1991): 1332-1337.
Regarding blood supply to the abdominal viscera:
a) List the abdominal structures supplied by the coeliac axis, superior mesenteric artery (SMA) and inferior mesenteric artery (IMA). (30% marks)
b) Explain which areas of intestine are more susceptible to mesenteric arterial ischaemia.
(10% marks)
c) Compare and contrast the risk factors, clinical characteristics, treatment, and prognosis of mesenteric arterial ischaemia (occlusive and non-occlusive) and mesenteric venous thrombosis.
(60% marks)
Not available.
a) The structures supplied by each major branch are:
b) Areas of intestine which are more susceptible to mesenteric arterial ischaemia:
c)
The clinical scenario is said to differentiate the causes of mesenteric ischaemia, as the clinical findings are typically similar for all of them:
And if you wait long enough, everything starts looking like severe septic shock and multiorgan system failure. In short, the abdomen is usually the same on examination, and the way you come to the conclusion that this must be embolic or venous, is by discovering that the patient is in AF, or has recently had a bowel resection for a colonic carcinoma, or something along those lines. Because of this, the risk factors and clinical characteristics were rolled together in this tabulated response.
Type | Risk factors and clinical characteristics |
Treatment | Prognosis |
Occlusive mesenteric ischaemia |
History of:
Also:
|
|
|
Non-occlusive mesenteric ischaemia |
|
|
|
Venous mesenteric ischaemia |
|
|
|
Acosta, Stefan, and Martin Björck. "Modern treatment of acute mesenteric ischaemia." British Journal of Surgery 101.1 (2014).
Schofield, Nick, et al. "Acute mesenteric ischaemia." Journal of the Intensive Care Society 15.3 (2014): 226-230.
Clair, Daniel G., and Jocelyn M. Beach. "Mesenteric ischemia." New England Journal of Medicine 374.10 (2016): 959-968. (pdf)
Pecoraro, Felice, et al. "Chronic mesenteric ischemia: critical review and guidelines for management." Annals of vascular surgery 27.1 (2013): 113-122.
Mastoraki, Aikaterini, et al. "Mesenteric ischemia: pathogenesis and challenging diagnostic and therapeutic modalities." World journal of gastrointestinal pathophysiology 7.1 (2016): 125.
Amini, Afshin, and Shivaraj Nagalli. "Bowel ischemia." StatPearls [Internet] (2020).
Bala, Miklosh, et al. "Acute mesenteric ischemia: guidelines of the World Society of Emergency Surgery." World Journal of Emergency Surgery 12.1 (2017): 1-11.
Hmoud, Bashar, Ashwani K. Singal, and Patrick S. Kamath. "Mesenteric venous thrombosis." Journal of clinical and experimental hepatology 4.3 (2014): 257-263.
Park, Woosup M., et al. "Contemporary management of acute mesenteric ischemia: factors associated with survival." Journal of vascular surgery 35.3 (2002): 445-452.
Leone, Marc, et al. "Outcome of acute mesenteric ischemia in the intensive care unit: a retrospective, multicenter study of 780 cases." Intensive care medicine 41.4 (2015): 667-676.